Tax Accounting test 1 Chapter 1-5, Income tax test 2

¡Supera tus tareas y exámenes ahora con Quizwiz!

Allowing for the overall limitation (50% reduction for meals), which of the following trips, if any, will qualify for the travel expense deduction? a. Dr. Jones, a self-employed general dentist, attends a two-day seminar on developing a dental practice. b. Dr. Brown, a self-employed surgeon, attends a two-day seminar on financial planning. c. Paul, a romance language high school teacher, spends summer break in France, Portugal, and Spain improving his language skills. d. Myrna went on a two-week vacation in Boston. While there, she visited her employer's home office to have lunch with former co-workers. e. All of these

A

Which, if any, of the following factors is not a characteristic of independent contractor status? a. Work-related expenses are reported on Form 2106. b. Receipt of a Form 1099 reporting payments received. c. Workplace fringe benefits are not available. d. Services are performed for more than one party. e. None of these.

A

Teal company is an accrual basis taxpayer. On December 1, 2018, a customer paid for an item that was on hand, but the customer wanted the item delivered in early January 2019. Teal delivered the item on January 4, 2019. Teal included the sale in its 2018 income for financial accounting purposes. a. Teal must recognize the income in 2018. b. Teal must recognize the income in the year title to the goods passed to the customer, as determined under the state laws in which the store is located. c. Teal can elect to recognize the income in either 2018 or 2019. d. Teal must recognize the income in 2019. e. None of these.

ANSWER: a RATIONALE: An accrual method taxpayer generally obtains no more deferral for tax than is allowed for financial reporting purposes.

Which, if any, of the statements regarding the standard deduction is correct? a. Some taxpayers may qualify for two types of standard deductions. b. The standard deduction is not available to taxpayers who are dependents. c. The standard deduction may be taken as a for AGI deduction. d. The basic standard deduction is indexed for inflation but the additional standard deduction is not. e. None of these.

ANSWER: a RATIONALE: An example of two additional standard deductions would be a taxpayer who is age 65 (or older) and blind (choice a.). Both the basic and the additional standard deductions are subject to indexation (choice d.). Dependents are allowed a limited standard deduction (choice b.). The standard deductions are in lieu of deductions from AGI—not for AGI (choice c.).

Jim and Nora, residents of a community property state, were married in early 2017. Late in 2017 they separated, and in 2018 they were divorced. Each earned a salary, and they received income from community owned investments in all relevant years. They filed separate returns in 2017 and 2018. a. In 2018, Nora must report only her salary and one-half of the income from community property on her separate return. b. In 2018, Nora must report on her separate return one-half of the Jim and Nora salary and one-half of the community property income. c. In 2018 Nora must report on her separate return one-half of the Jim and Nora salary for the period they were married as well as one-half of the community property income and her income earned after the divorce. d. In 2018, Nora must report only her salary on her separate return. e. None of these

ANSWER: a RATIONALE: Because Jim and Nora lived apart for the entire year, she does not have to report one-half of Jim's salary on her separate return. She is required to report her share of the income from the community owned investments.

This year, Carol, a single taxpayer, purchased a vacation home for $400,000 using an equity debt of $350,000 on her principal residence. Carol has no other debt on her principal residence. Carol paid $16,000 of interest on the debt this year. How much of this interest is deductible assuming Carol itemizes her deductions? a. $0 b. $10,000 c. $16,000 d. $125,000 e. None of the above

ANSWER: a RATIONALE: Because the debt is not secured by the vacation home, it is equity debt on the principal residence which is not deductible

In 2018, Boris pays a $3,800 premium for high-deductible medical insurance for himself and his family. In addition, he contributes $3,400 to a Health Savings Account. Which of the following statements is true? a. If Boris is self-employed, he may deduct $7,200 as a deduction for AGI. b. If Boris is self-employed, he may deduct $3,400 as a deduction for AGI and may include the $3,800 premium when calculating his itemized medical expense deduction. c. If Boris is an employee, he may deduct $7,200 as a deduction for AGI. d. If Boris is an employee, he may include $7,200 when calculating his itemized medical expense deduction. e. None of the above.

ANSWER: a RATIONALE: Boris, who is self-employed, may deduct 100% of the premium ($3,800) as a deduction for AGI. He may also deduct the $3,400 HSA contribution as a deduction for AGI.

Barry and Larry, who are brothers, are equal owners in Chickadee Corporation. On July 1, 2018, each loans the corporation $10,000 at an annual interest rate of 10%. Both shareholders are on the cash method of accounting, while Chickadee Corporation is on the accrual method. All parties use the calendar year for tax purposes. On June 30, 2019, Chickadee repays the loans of $20,000 together with the specified interest of $2,000. How much of the interest can Chickadee Corporation deduct in 2018? a. $0 b. $500 c. $1,000 d. $2,000 e. None of the above

ANSWER: a RATIONALE: Chickadee Corporation can deduct interest expense of $2,000 in 2019 and $0 in 2018. Under § 267, Barry and Larry are regarded as related to the corporation. Consequently, the deductibility must await actual payment (in 2019).

Which of the following is a deduction for AGI? a. Contribution to a traditional IRA. b. Roof repairs to a personal use home. c. Safe deposit box rental fee in which stock certificates are stored. d. Property tax on personal residence. e. All of the above.

ANSWER: a RATIONALE: Choice c. and d. are deductions from AGI. Choice b. is not deductible.

If a residence is used primarily for personal use (rented for fewer than 15 days per year), which of the following is correct? a. No income is included in AGI. b. No expenses are deductible. c. Expenses must be allocated between rental and personal use. d. Only a. and b. are correct. e. a., b., and c. are correct

ANSWER: a RATIONALE: Expenses that would otherwise be deductible (e.g., property taxes and interest on mortgage of personal residence) can be claimed (choice b.).

Under the actual expense method, which, if any, of the following expenses will not be allowed? a. Parking fines incurred during business use of a car. b. Interest expense on a car loan (taxpayer is self-employed). c. Auto insurance. d. Auto club dues. e. None of the above.

ANSWER: a RATIONALE: Fines and penalties are not deductible

In contrasting the reporting procedures of employees and self-employed persons regarding job-related transactions, which of the following items involve self-employed? a. Schedule C of Form 1040. b. Form W-4. c. Form W-2. d. Schedule A of Form 1040. e. None of these.

ANSWER: a RATIONALE: Form W-4 is used by employees to assist their employer(s) withhold taxes appropriately (choice b.). Schedule C is used to list the job-related income and expenses of self-employed taxpayers (choice a.). Form W-2 reports the wages earned by employees (choice c.). Income earned by self-employed persons is often shown on Form 1099. The results of Form 2106 (see choice b.) are transferred to Schedule A (choice d.)

As a general rule: I. Income from property is taxed to the person who owns the property. II. Income from services is taxed to the person who earns the income. III. The assignee of income from property must pay tax on the income. IV. The person who receives the benefit of the income must pay the tax on the income. a. Only I and II are true. b. Only III and IV are true. c. I, II, and III are true, but IV is false. d. I, II, III, and IV are true. e. None of these is true.

ANSWER: a RATIONALE: III is false because a person who has the right to income (the assignor) but assigns the rights to another must pay the tax on the income. IV is false because, for example, the assignee of income receives the benefit, but the assignor has the right to the income and therefore must pay the tax on that income

Which, if any, of the following is a deduction for AGI? a. Contributions to a traditional Individual Retirement Account. b. Child support payments. c. Funeral expenses. d. Loss on the sale of a personal residence. e. Medical expenses.

ANSWER: a RATIONALE: IRA contributions (choice a.) are deductions for AGI. Child support payments (choice b.), funeral expenses (choice c.), and personal capital losses (choice d.) are nondeductible items. Medical expenses are deductions from AGI.

On a particular Saturday, Tom had planned to paint a room in his house, but his employer gave him the opportunity to work that day. If Tom works, he must hire a painter for $120. For Tom to have a positive cash flow from working and hiring the painter: a. Tom must earn more than $158 if he is in the 24% marginal tax bracket. b. Tom must earn at least $158 if he is in the 32% marginal tax bracket. c. Tom must earn at least $140 if he is in the 24% marginal tax bracket. d. Tom must earn at least $120 if he is in the 12% marginal tax bracket. e. None of these.

ANSWER: a RATIONALE: If Tom earns $158, he will have after-tax pay of $120 [(1 - .24)($158)], which is equal to what he must pay the painter

During 2018, Enrique had the following transactions: Salary $70,000 Interest income on Xerox bonds 2,000 Inheritance from uncle 40,000 Contribution to traditional IRA 5,500 Capital losses 2,500 Enrique's AGI is: a. $62,000. b. $64,000. c. $67,000. d. $102,000. e. $104,000.

ANSWER: b RATIONALE: $70,000 (salary) + $2,000 (interest) - $5,500 (IRA contribution) - $2,500 (capital losses) = $64,000. The inheritance is a nontaxable exclusion. The capital losses are deductible.

Regarding the Tax Tables applicable to the Federal income tax, which of the following statements is correct? a. For any one year, the Tax Tables are issued by the IRS after the Tax Rate Schedules. b. The Tax Tables will always yield the same amount of tax as the Tax Rate Schedules. c. Taxpayers can elect as to whether the use the Tax Tables or the Tax Rate Schedules. d. The Tax Tables can be used by an estate but not by a trust. e. No correct answer given.

ANSWER: a RATIONALE: In any one year, the Tax Rate Schedules are issued after the Tax Tables (choice a.). Because of the way the Tables are structured, a minor variation in the tax can occur (choice b.). Where applicable, the Schedules must be used and taxpayers do not have an election (choice c.). The Tax Tables cannot be used by either an estate or trust (choice d.)

Joyce, age 40, and Sam, age 42, who have been married for seven years, are both active participants in qualified retirement plans. Their total AGI for 2018 is $130,000. Each is employed and earns a salary of $65,000. What are their combined deductible contributions to traditional IRAs? a. $0 b. $3,000 c. $4,000 d. $8,000 e. None of the above

ANSWER: a RATIONALE: Joyce and Sam may contribute a total of $11,000 ($5,500 each) to a traditional IRA, but because their AGI exceeds the phaseout ceiling of $121,000 in 2018, $0 is deductible.

Kyle, whose wife died in December 2015, filed a joint tax return for 2015. He did not remarry, but has continued to maintain his home in which his two dependent children live. What is Kyle's filing status in 2018? a. Head of household b. Surviving spouse c. Single d. Married filing separately e. None of these

ANSWER: a RATIONALE: Kyle, who filed a joint return in 2015, was entitled to file as a surviving spouse in 2016 and 2017. In 2018, he will be entitled to file as a head of household.

Velma and Bud divorced. Velma's attorney fee of $5,000 is allocated as follows: General representation in obtaining the divorce $1,500 Services in obtaining custody of the child 900 Services in settlement of martial property 600 Determining the tax consequences of: Dependency deduction for child 700 Tax Consequences of property settlement 1,300 Of the $5,000 Velma pays to her attorney in 2018, the amount she may deduct as an itemized deduction is: a. $0. b. $700. c. $2,000. d. $5,000. e. None of the above.

ANSWER: a RATIONALE: Legal fees that relate to tax advice are no longer deductible.

Louise works in a foreign branch of her employer's business. She earned $5,000 per month throughout the relevant period. Which of the following is correct: a. If Louise worked in the foreign branch from May 1, 2017 until October 31, 2018, she may exclude $40,000 from gross income in 2017 and exclude $50,000 in 2018. b. If Louise worked in the foreign branch from May 1, 2017 until October 31, 2018, she cannot exclude anything from gross income because she was not present in the country for 330 days in either year. c. If Louise began work in the foreign country on May 1, 2017, she must work through November 30, 2018 in order to exclude $55,000 from gross income in 2018 but none in 2017. d. Louise will not be allowed to exclude any foreign earned income because she made less than $103,900. e. None of these.

ANSWER: a RATIONALE: Louise was in the foreign country for the requisite number of days, at least 330 days out of 365 days. Therefore, the foreign earned income exclusion applies for all of the time she worked in the foreign country. The amount earned is less than the statutory annual ceiling. Choice d. is incorrect because the annual limit is the amount, if earned, that can be excluded

Mark a calendar year taxpayer, purchased an annuity for $50,000 in 2016. The annuity was to pay him $3,000 on the first day of each year, beginning in 2016, for the remainder of his life. Mark's life expectancy at the time he purchased the annuity was 20 years. In 2018 Mark developed a deadly disease, and doctors estimated that he would live for no more than 24 months. a. If Mark dies in 2019, a loss can be claimed on his final return for his unrecovered cost of the annuity. b. If Mark dies in 2019, his returns for the two previous years can be amended to allocate the entire cost of the annuity to the years in which he received payments and reported gross income. c. If Mark is still alive at the end of 2018, he is not required to recognize any gross income because of his terminal illness. d. If Mark is still alive in 2038, his recovery of capital for that year is $500. e. None of these.

ANSWER: a RATIONALE: Mark's final return can report a loss in 2019 because he did not recover all of his cost of the annuity prior to his death. Answer d. is incorrect because Mark would have recovered his entire cost of the contract in the first 20 years under the contract, 2016-2035. Answers b. and c. are also incorrect.

Mike contracted with Kram Company, Mike's controlled corporation. Mike was a medical doctor and the contract provided that he would work exclusively for the corporation. No other doctor worked for the corporation. The corporation contracted to perform an operation for Rosa for $8,000. The corporation paid Mike $6,500 to perform the operation under the terms of his employment contract. a. Mike's gross income is $6,500. b. Mike must recognize the $8,000 gross income because he provided the service. c. Mike must recognize $8,000 gross income since the patient obviously wanted him to perform the operation. d. The Kram Company corporation's gross income is $1,500. e. None of these.

ANSWER: a RATIONALE: Mike is an employee of Kram Company. Thus, Kram Company is taxed on the $8,000 income from the services provided by Mike to the patient. Mike is taxed on the $6,500 of compensation received from Kram

Stuart owns 300 shares of Turquoise Corporation stock and 2,000 shares of Blue Corporation stock. During the year, Stuart received 150 shares of Turquoise as a result of a 1 for 2 stock split. The value of the shares received was $4,800. Stuart also received 100 shares of Blue Corporation stock as a result of a 5% stock dividend. Stuart did not have the option of receiving cash from Blue. The additional shares he received had a value of $7,200. Stuart's gross income from the receipt of the additional Turquoise and Blue shares is: a. $0. b. $4,800. c. $7,200. d. $12,000. e. None of these.

ANSWER: a RATIONALE: Neither the stock dividend nor the stock split is taxable

On January 1, 2008, Cardinal Corporation issued 5% 25-year bonds at par and used the $12,000,000 proceeds to finance the construction of a new plant. On January 1, 2018, the company acquired the bonds on the open market for $11,500,000. Assuming that Cardinal Corporation is neither bankrupt nor insolvent, the acquisition and retirement of the bonds results in which of the following: a. The company must recognize a $500,000 gain. b. The company can make an election to recognize a $500,000 gain or reduce the company's basis in the plant by $500,000. c. The company must recognize a $500,000 gain and increase the company's basis in the plant by $500,000. d. The company can amortize the $500,000 gain, recognizing income over the remaining life of the bonds. e. None of these.

ANSWER: a RATIONALE: None of the exceptions to income from discharge of indebtedness apply.

. Melba incurred the following expenses for her dependent daughter during the current year: Payment of principal on daughter's automobile loan $3,600 Payment of interest on above loan 2,900 Payment of daughter's property taxes 1,800 Payment of principal on daughter's personal residence loan 2,800 Payment of interest on daughter's personal residence loan 7,000 How much may Melba deduct in computing her itemized deductions? a. $0. b. $8,800. c. $11,700. d. $18,100. e. None of the above.

ANSWER: a RATIONALE: None of the items are incurred for the taxpayer's (Melba) benefit or as a result of the taxpayer's obligation

Andrew, who operates a laundry business, incurred the following expenses during the year. ∙ Parking ticket of $250 for one of his delivery vans that parked illegally. ∙ Parking ticket of $75 when he parked illegally while attending a rock concert in Tulsa. ∙ DUI ticket of $500 while returning from the rock concert. ∙ Attorney's fee of $600 associated with the DUI ticket. What amount can Andrew deduct for these expenses? a. $0. b. $250. c. $600. d. $1,425. e. None of the above.

ANSWER: a RATIONALE: None of these expenses are deductible. The $75 parking ticket, the $500 DUI ticket, and the $600 attorney fee are all personal expenses. The $250 parking ticket, although related to his laundry business, is not deductible because it is a violation of public policy.

In the current year, Jerry pays $8,000 to become a charter member of Mammoth University's Athletic Council. The membership ensures that Jerry will receive choice seating at all of Mammoth's home basketball games. Also this year, Jerry pays $2,200 (the regular retail price) for season tickets for himself and his wife. For these items, how much qualifies as a charitable contribution? a. $0 b. $6,400 c. $8,000 d. $10,200 e. None of the above

ANSWER: a RATIONALE: Prior to the TCJA of 2017, 80 percent of the $8,000 would be a charitable contribution. With the TCJA of 2017, no deduction is allowed for seating rights.

Olaf was injured in an automobile accident and received $25,000 for his physical injury, $50,000 for his loss of income, and $10,000 punitive damages. As a result of the award, the amount Olaf must include in gross income is: a. $10,000. b. $50,000. c. $60,000. d. $85,000. e. None of these.

ANSWER: a RATIONALE: Punitive damages are never excluded from gross income

Theresa sued her former employer for age, race, and gender discrimination. She claimed $200,000 in damages for loss of income, $300,000 for emotional harm, and $500,000 in punitive damages. She settled the claim for $700,000. As a result of the settlement, Theresa must include in gross income: a. $700,000. b. $500,000. c. $490,000 [($700,000/$1,000,000) × $700,000]. d. $0. e. None of these.

ANSWER: a RATIONALE: The damages did not arise out of a physical personal injury; therefore, none of the amount received can be excluded from gross income

Rex, a cash basis calendar year taxpayer, runs a bingo operation which is illegal under state law. During 2018, a bill designated H.R. 9 is introduced into the state legislature which, if enacted, would legitimize bingo games. In 2018, Rex had the following expenses: Operating expenses in conducting bingo games $247,000 Payoff money to state and local police 24,000 Newspaper ads supporting H.R. 9 3,000 Political contributions to legislators who support H.R. 9 8,000 Of these expenditures, Rex may deduct: a. $247,000. b. $250,000. c. $258,000. d. $282,000. e. None of the above.

ANSWER: a RATIONALE: Rex can deduct only the $247,000 of operating expenses

Hugh, a self-employed individual, paid the following amounts during the year: Real estate tax on Iowa residence $3,800 State income tax 1,700 Real estate taxes on a vacation home 2,100 Gift tax paid on gift to daughter 1,200 State sales taxes 1,750 State occupational license fee 300 Property tax on value of his automobile (used 100% for business) 475 What is the maximum amount Hugh can claim as taxes in itemizing deductions from AGI? a. $7,650 b. $8,850 c. $9,625 d. $10,000 e. None of the above

ANSWER: a RATIONALE: State sales taxes ($1,750) are more than the state income tax ($1,700); so Hugh should choose to deduct the sales tax rather than the state income tax. The state occupational license fee ($300) and the tax on his business use auto ($475) are deductible for AGI as business expenses. The state sales tax ($1,850) and the real estate taxes ($3,800 + $2,100) are deductible as itemized deductions. The gift tax is not deductible. The total itemized deductions are $7,650 ($3,800 + $2,100 + $1,750).

Which of the following is not a related party for constructive ownership purposes under § 267? a. The taxpayer's aunt. b. The taxpayer's brother. c. The taxpayer's grandmother. d. A corporation owned more than 50% by the taxpayer. e. None of the above.

ANSWER: a RATIONALE: Taxpayer's aunt is not a related party under § 267

Daniel purchased a bond on July 1, 2018, at par of $10,000 plus accrued interest of $300. On December 31, 2018, Daniel collected the $600 interest for the year. On January 1, 2019, Daniel sold the bond for $10,200. a. Daniel must recognize $300 interest income for 2018 and a $200 gain on the sale of the bond in 2019. b. Daniel must recognize $600 interest income for 2018 and a $200 gain on the sale of the bond in 2019. c. Daniel must recognize $600 interest income for 2018 and a $100 loss on the sale of the bond in 2019. d. Daniel must recognize $300 interest income for 2018 and a $100 loss on the sale of the bond in 2019. e. None of these.

ANSWER: a RATIONALE: The $600 collected consists of $300 of gross income for the interest earned from July 1 through December 31 and $300 of accrued interest that was purchased. The cost of the bond was $10,000; thus Daniel has a $200 gain ($10,200 - $10,000) on the sale

Under the deemed substantiation method of accounting for expenses, what is the maximum amount taxpayers are allowed as a deduction without being required to substantiate the amount of the expenses? a. The appropriate Federal per diem amount. b. $75 per day. c. All expenses up to $25 per day. d. The per diem rate established by the state in which they live. e. None of the above.

ANSWER: a RATIONALE: The Federal per diem rate is the amount deemed substantiated.

On January 1, Father (Dave) loaned Daughter (Debra) $100,000 to purchase a new car and to pay off college loans. There were no other loans outstanding between Dave and Debra. The relevant Federal rate on interest was 6 percent. The loan was outstanding for the entire year. a. If Debra has $15,000 of investment income, Dave must recognize $6,090 of imputed interest income. b. Dave must recognize $6,090 of imputed interest income regardless of the amount of Debra's investment income. c. Debra must recognize $6,090 of imputed interest income. d. Debra must recognize $6,090 of imputed interest income if Dave has at least $6,090 of investment income. e. None of these.

ANSWER: a RATIONALE: The calculated imputed interest is $6,090 [($100,000 × 6% × .5) + ($103,000 × 6% × .5)]. Under the $100,000 exemption for below-market loans, Dave must recognize $6,090 of imputed interest income because Debra has investment income in excess of the calculated imputed interest income

Theresa, a cash basis taxpayer, purchased a bond on July 1, 2014, for $10,000, plus $400 of accrued interest. The bond paid $800 of interest each December 31. On March 31, 2018, she sold the bond for $9,800, which included $200 of accrued interest. a. Theresa has $200 interest income and a $400 loss from the bond in 2018. b. Theresa has $200 interest income and a $200 gain from the bond in 2018. c. Theresa has a $100 loss from the sale of the bond and no interest income. d. Theresa's loss on the sale of the bond is $600. e. None of these.

ANSWER: a RATIONALE: The cost of the bond was $10,000 and the proceeds from the sale were $9,600 ($9,800 - $200 accrued interest). Therefore, Theresa had a $400 ($9,600 - $10,000) loss from the sale, and $200 of interest income.

Evaluate the following statements: I. De minimis fringe benefits are those that are so immaterial that accounting for them is impractical. II. De minimis fringe benefits are subject to strict anti-discrimination requirements. III. Generally, a fringe benefit of less than $50 is considered de minimis and can be excluded from gross income. a. Only I is true. b. Only III is true. c. Only I and III are true. d. I, II, and III are true. e. None of these.

ANSWER: a RATIONALE: The de minimis exemption is not subject to a nondiscrimination requirement because the amount is too small to make it worthwhile to account for the items. It follows that it would not be worthwhile to apply additional rules.

The Royal Motor Company manufactures automobiles. Non-management employees of the company can buy a new automobile for Royal's cost plus 2%. The automobiles are sold to dealers at cost plus 20%. Generally, management employees of Local Dealer, Inc., are allowed to buy a new automobile from the company at the dealer's cost. Which of the following statements is correct? a. The non-management employees who buy automobiles at a discount are not required to recognize income from the purchase. b. None of the employees who take advantage of the fringe benefits described above are required to recognize income. c. Employees of Royal are required to recognize as gross income 18% (20% - 2%) of the cost of the automobile purchased. d. All of these. e. None of these.

ANSWER: a RATIONALE: The discounts for the non-management employees of Royal Motor Company qualify for exclusion treatment as a "qualified employee discount." Management employees of Local Dealer, Inc. discount is discriminatory, therefore exclusion treatment is denied.

Which, if any, of the following is an advantage of using the simplified method for determining the office in the home deduction? a. No depreciation on the personal residence has to be computed. b. The exclusive use requirement does not have to be met. c. Allows the expense to be classified as a deduction for AGI. d. Can also be used for a residence that is rented (not owned) by the taxpayer. e. None of these.

ANSWER: a RATIONALE: The exclusive use requirement (choice b.) always has to be met for an office in the home deduction to be available. The method chosen for arriving at the deduction has no relevance to how it is classified (i.e., deduction for or from AGI)--choice c. Either the simplified method or the regular method can be used for rented property (choice d.).

Peggy is an executive for the Tan Furniture Manufacturing Company. Peggy purchased furniture from the company for $9,500, the price Tan ordinarily would charge a wholesaler for the same items. The retail price of the furniture was $12,500, and Tan's cost was $9,000. The company also paid for Peggy's parking space in a garage near the office. The parking fee was $600 for the year. All employees are allowed to buy furniture at a discounted price comparable to that charged to Peggy. However, the company does not pay other employees' parking fees. Peggy's gross income from the above is: a. $0. b. $600. c. $3,500. d. $4,100. e. None of these.

ANSWER: a RATIONALE: The furniture purchases were under a "qualified employee discount" plan, but the exclusion is limited to the employer's gross profit. Because Peggy purchased the furniture for $9,500 when the employer's cost was $9,000, she is not required to include anything in gross income as a result of the purchase. The parking space with a value of $600 is a qualified transportation fringe and is not required to be available to all employees (i.e., can be provided on a discriminatory basis).

Darryl, a cash basis taxpayer, gave 1,000 shares of Copper Company common stock to his daughter on September 29, 2018. Copper Company is a publicly held company that has declared a $2.00 per share dividend on September 30th every year for the last 20 years. Just as Darryl had expected, Copper Company declared a $2.00 per share dividend on September 30th, payable on October 15th, to stockholders of record as of October 10th. The daughter received the $2,000 dividend on October 18, 2018. a. The daughter must recognize the income because she owned the stock when the dividend was declared and she received the $2,000. b. Darryl must recognize the income of $2,000 because the purpose of the gift was to avoid taxes. c. Darryl must recognize $1,500 of the dividend because he owned the stock for three-fourths of the year. d. Darryl must recognize the $2,000 dividend as his income because he constructively received the dividend. e. None of these.

ANSWER: a RATIONALE: The gift of the stock is made prior to the declaration date

Adam repairs power lines for the Egret Utilities Company. He is generally working on a power line during the lunch hour. He must eat when and where he can and still get his work done. He usually purchases something at a convenience store and eats in his truck. Egret reimburses Adam for the cost of his meals. a. Adam must include the reimbursement in his gross income. b. Adam can exclude the reimbursement from his gross income since the meals are provided for the convenience of the employer. c. Adam can exclude the reimbursement from his gross income because he eats the meals on the employer's business premises (the truck). d. Adam may exclude from his gross income the difference between what he paid for the meals and what it would have cost him to eat at home. e. None of these.

ANSWER: a RATIONALE: The meals do not qualify for the meals and lodging exclusion because the meals are not furnished by the employer.

Jerry purchased a U.S. Series EE savings bond for $744. The bond has a maturity value in 10 years of $1,000 and yields 3% interest. This is the first Series EE bond that Jerry has ever owned. a. Jerry can defer the interest income until the bond matures in 10 years. b. Jerry must report ($1,000 - $744)/10 = $25.60 interest income each year he owns the bond. c. The interest on the bonds is exempt from Federal income tax. d. Jerry can report all of the $256 as a capital gain in the year it matures. e. None of these.

ANSWER: a RATIONALE: The original issue discount (OID) on the Series EE bonds is not subject to the OID rules. However, the income is interest, rather than gain from the sale of a capital asset.

Randy is the manager of a motel. As a condition of his employment, Randy is required to live in a room on the premises so that he would be there in case of emergencies. Randy considered this a fringe benefit, since he would otherwise be required to pay $800 per month rent. The room that Randy occupied normally rented for $70 per night, or $2,100 per month. On the average, 90% of the motel rooms were occupied. As a result of this rent-free use of a room, Randy is required to include in gross income. a. $0. b. $800 per month. c. $2,100 per month. d. $1,890 ($2,100 × .90). e. None of these.

ANSWER: a RATIONALE: The room qualifies for the § 119 lodging exclusion.

In terms of the tax formula applicable to individual taxpayers, which, if any, of the following statements is correct? a. In arriving at taxable income, a taxpayer must choose between the standard deduction and itemized deductions. b. In arriving at AGI, personal and dependency exemptions are subtracted from gross income. c. In arriving at taxable income, a taxpayer must choose between the standard deduction and the deduction for qualified business income. d. The tax formula does not apply if a taxpayer elects to claim the standard deduction. e. None of these.

ANSWER: a RATIONALE: The standard deduction is not an alternative; it is a component of the tax formula (choice d.). Personal and dependency exemptions have been suspended from 2018 through 2025 and, if allowed, are subtracted after AGI is determined (choice b.). The choice is between the standard deduction and deductions from AGI (choice a.)

Travis and Andrea were divorced in 2016. Their only marital property consisted of a personal residence (fair market value of $400,000, cost of $200,000), and publicly-traded stocks (fair market value of $800,000, cost basis of $500,000). Under the terms of the divorce agreement, Andrea received the personal residence and Travis received the stocks. In addition, Andrea was to receive $50,000 for eight years. I. If the $50,000 annual payments are to be made to Andrea or her estate (if she dies before the end of the eight years), the payments will qualify as alimony. II. Andrea has a taxable gain from an exchange of her one-half interest in the stocks for Travis' one-half interest in the house and cash. III. If Travis sells the stocks for $900,000, he must recognize a $400,000 gain. a. Only III is true. b. Only I and III are true. c. Only I and II are true. d. I, II, and III are true. e. None of these are true

ANSWER: a RATIONALE: To qualify as alimony, the cash payments must cease upon the death of the payee; therefore, option I is incorrect. The basis of the marital property received by a former spouse is the same as the former married couple's basis. Therefore, Travis' basis in the stocks was $500,000, and when they were sold, Travis recognized a $400,000 gain (option III).

Martha participated in a qualified tuition program for the benefit of her son. She invested $6,000 in the fund. Four years later her son withdrew $8,000, the entire balance in the program, to pay his college tuition. a. Martha is not required to include the $2,000 ($8,000 - $6,000) in her gross income when the funds are used to pay the tuition. b. Martha's son must include the $2,000 ($8,000 - $6,000) in his gross income when the funds are used to pay the tuition. c. Martha must include $8,000 in her gross income. d. Martha's son must include $8,000 in his gross income. e. None of these

ANSWER: a RATIONALE: Under a qualified tuition program, neither the beneficiary of the income (the son) nor the owner (Martha) of the property includes the earnings in gross income as long as the funds are used to pay qualified tuition.

Pat gave 5,000 shares of stock in Coyote Corporation (a publicly traded corporation) to her church (a qualified charitable organization) in the current year. The stock was worth $180,000 and she had acquired it as an investment four years ago at a cost of $120,000. She reported AGI of $300,000 for the year. In completing her current income tax return, how much is her current-year charitable contribution deduction? a. $90,000 b. $120,000 c. $150,000 d. $180,000 e. None of the above

ANSWER: a RATIONALE: Under the general rule concerning capital gain property, Pat can deduct $90,000 (30% of $300,000 AGI) in the current year. The remaining $90,000 [$180,000 (fair market value of the Coyote Corporation stock) - $90,000 (current deduction)] can be carried over for up to five years, subject to the 30% limitation when used.

Wayne owns a 30% interest in the capital and profits of Emerald Company (a calendar year partnership). For tax year 2018, the partnership earned revenue of $900,000 and had operating expenses of $660,000. During the year, Wayne withdrew from the partnership a total of $90,000. He also invested an additional $30,000 in the partnership. For 2018, Wayne's gross income from the partnership is: a. $72,000. b. $90,000. c. $132,000. d. $162,000.

ANSWER: a RATIONALE: Wayne must report as his gross income 30% of the partnership profits, 30% × ($900,000 - $660,000) = $72,000. The $90,000 is a return of capital. The $30,000 is a contribution to capital.

Millie, age 80, is supported during the current year as follows: Percent of Support Weston (a son) 20% Faith (a daughter) 35% Jake (a cousin) 25% Brayden (unrelated close family friend) 20% During the year, Millie lives in an assisted living facility. Under a multiple support agreement, indicate which parties can qualify to claim Millie as a dependent. a. Weston and Faith. b. Faith. c. Weston, Faith, Jake, and Brayden. d. Faith, Jake, and Brayden. e. None of these.

ANSWER: a RATIONALE: Weston and Faith are the only persons who appear to qualify (choice a.). They qualify because they contribute more than 10% of the support. Jake does not qualify because he satisfies neither the relationship nor member of the household tests. (This eliminates choices c. and d.) Brayden does not meet the relationship test, and he does not satisfy the member of the household test. For a multiple support agreement to be effective, the qualifying individuals collectively must provide greater than 50% of the support.

Matilda works for a company with 1,000 employees. The company has a hospitalization insurance plan that covers all employees. However, the employee must pay the first $3,000 of his or her medical expenses each year. Each year, the employer contributes $1,500 to each employee's health savings account (HSA). Matilda's employer made the contributions in 2017 and 2018, and the account earned $100 interest in 2018. At the end of 2018, Matilda withdrew $3,100 from the account to pay the deductible portion of her medical expenses for the year and other medical expenses not covered by the hospitalization insurance policy. As a result, Matilda must include in her 2018 gross income: a. $0. b. $100. c. $1,600. d. $3,100. e. None of these.

ANSWER: a RATIONALE: With a health savings account (HSA), the employee is not taxed when the money is contributed, as income is earned in the account, or when amounts are withdrawn to pay for medical expenses.

During the year, Sophie (a self-employed marketing consultant) went from Omaha to Lima (Peru) on business. She spent four days on business, two days on travel, and four days on vacation. Disregarding the vacation costs, Sophie's expenses are: Air fare $3,000 Lodging 800 Meals 600 Entertainment 400 Sophie's deductible expenses are: a. $4,300. b. $2,900. c. $2,800. d. $2,500. e. None of these.

ANSWER: b RATIONALE: $1,800 [60% (6 days business/10 day trip) × $3,000 (air fare)] + $800 + $300 [50% x $600] = $2,900. The air fare has to be allocated as Sophie did not meet either the seven days (or less) or less than 25% personal use exceptions for foreign travel.

In Lawrence County, the real property tax year is the calendar year. The real property tax becomes a personal liability of the owner of real property on January 1 in the current real property tax year (assume this year is not a leap year). The tax is payable on June 1. On May 1, Reggie sells his house to Dana for $350,000. On June 1, Dana pays the entire real estate tax of $7,950 for the year ending December 31. Assuming that Reggie itemizes his deductions and the $10,000 limit on state and local taxes does not apply, how much of the property taxes may Reggie deduct? a. $0 b. $2,614 c. $2,625 d. $7,950 e. None of the above

ANSWER: b RATIONALE: $2,614. Under § 164(d), 120/365 (January 1 - April 30) × $7,950 = $2,614 is apportioned to Reggie.

Ralph made the following business gifts during the year. To Robert (a key client) at Christmas $50 To Angel (Robert's 8-year old daughter) on her birthday 20 To Art (Ralph's secretary) on his birthday ($3 was for gift wrapping) 30 To Paige (Ralph's boss) at Christmas 40 Presuming proper substantiation, Ralph's deduction is: a. $0. b. $53. c. $73. d. $78. e. $98.

ANSWER: b RATIONALE: $25 (Robert) + $28 (Art) = $53. The gift to Angel counts as a gift to Robert (whose $25 limit already has been reached). Gifts to superiors (Paige) cannot be deducted.

Karen, a calendar year taxpayer, made the following donations to qualified charitable organizations during the year: Basis Fair Market Value Cash donation to State University $30,000 $ 30,000 Unimproved land to the City of Terre Haute, Indiana 70,000 210,000 The land had been held as an investment and was acquired 4 years ago. Shortly after receipt, the City of Terre Haute sold the land for $210,000. Karen's AGI is $450,000. The allowable charitable contribution deduction this year is: a. $100,000. b. $165,000. c. $225,000. d. $240,000. e. None of the above

ANSWER: b RATIONALE: $30,000 (cash) + $135,000 (30% × $450,000 AGI) = $165,000. The capital gain property is limited to 30% of $450,000 AGI, or $135,000. The carryover to the next five years is $75,000 [$210,000 (FMV of the land) - $135,000 (deduction allowed for the current year)].

During 2018, Marvin had the following transactions: Salary $50,000 Bank loan (proceeds used to buy personal auto) 10,000 Alimony paid (divorce was finalized in 2010). 12,000 Child support paid 6,000 Gift from aunt 20,000 Marvin's AGI is:, a. $32,000. b. $38,000. c. $44,000. d. $56,000. e. $64,000.

ANSWER: b RATIONALE: $50,000 (salary) - $12,000 (alimony) = $38,000. The gift is an exclusion while the child support is nondeductible. Amounts borrowed are not income.

Which, if any, of the following expenses are deductible? a. Safety shoes purchased by an plumber employed by a company. b. Bottled water purchased by a gig driver for passengers. c. Unreimbursed employee expenses. d. Tax return preparation fee paid by a non-employed retiree. e. None of these.

ANSWER: b RATIONALE: All employee expenses, unless reimbursed pursuant to an adequate accounting, are not deductible (choices a. and c.). Tax preparation fees, unless related to a business, are not deductible.

During 2018, Lisa (age 66) furnished more than 50% of the support of the following persons: ∙ Lisa's current husband who has no income and is not claimed by someone else as a dependent. ∙ Lisa's stepson (age 19) who lives with her and earns $6,000 as a dance instructor. He dropped out of school a year ago. ∙ Lisa's ex-husband who does not live with her. The divorce occurred two years ago. ∙ Lisa's former brother-in-law who does not live with her. Presuming all other dependency tests are met, on a separate return how many dependents may Lisa claim? a. Two b. Three c. Four d. Five e. None of these

ANSWER: b RATIONALE: All of the persons listed except the ex-husband meet either the relationship or member of the household tests. The current husband qualifies as he has no income and is not claimed as a dependent by someone else. The stepson does not avoid the gross income limitation of a qualifying relative. He is not a qualifying child under 19 years of age.

Turquoise Company purchased a life insurance policy on the company's chief executive officer, Joe. After the company had paid $400,000 in premiums, Joe died and the company collected the $1.5 million face amount of the policy. The company also purchased group term life insurance on all its employees. Joe had included $16,000 in gross income for the group term life insurance premiums. Joe's widow, Rebecca, received the $100,000 proceeds from the group term life insurance policy. a. Rebecca can exclude the life insurance proceeds of $100,000, but Turquoise Company must include $1,100,000 ($1,500,000 - $400,000) in gross income. b. Turquoise Company and Rebecca can exclude the life insurance proceeds of $1,500,000 and $100,000, respectively, from gross income. c. Turquoise Company can exclude $1,100,000 ($1,500,000 - $400,000) from gross income, but Rebecca must include $84,000 in gross income. d. Turquoise Company must include $1,100,000 ($1,500,000 - $400,000) in gross income and Rebecca must include $100,000 in gross income. e. None of these

ANSWER: b RATIONALE: All of the proceeds qualify for the life insurance exclusion because the payments were received as a result of the death of the insured.

Which of the following items, if any, is deductible? a. Parking expenses incurred in connection with jury duty—taxpayer is a dentist. b. Substantiated gambling losses (not in excess of gambling winnings) from state lottery. c. Contributions to mayor's reelection campaign. d. Speeding ticket incurred while on business. e. Premiums paid on personal life insurance policy.

ANSWER: b RATIONALE: All other choices are not deductible

With respect to the prepaid income from services, which of the following is true? a. The treatment of prepaid income is the same for tax and financial accounting for both cash and accrual basis taxpayers. b. A cash basis taxpayer must report all of the income in the year received. c. An accrual basis taxpayer can spread the income over the period services are to be provided if all of the services will be completed within three years following the year of receipt. d. An accrual basis taxpayer can spread the income over the period services are to be provided on a contract for three years or less. e. None of these.

ANSWER: b RATIONALE: Answer a. is incorrect because the tax treatment is based on the income tax provisions, whereas the financial accounting treatment is based on generally accepted accounting principles. Answers c and d are incorrect. An accrual method taxpayer may adopt a method for prepaid service income to report the same amount on its tax return as is reported on its financial statements for the year of receipt with the balance reported on the subsequent year's tax return (§ 451(c))

In December 2018, Todd, a cash basis taxpayer, paid $1,200 of fire insurance premiums for the calendar year 2019 on a building he held for rental income. Todd deducted the $1,200 of insurance premiums on his 2018 tax return. He had $150,000 of taxable income that year. On June 30, 2019, he sold the building and, as a result, received a $500 refund on his fire insurance premiums. As a result of the above: a. Todd should amend his 2018 return and claim $500 less insurance expense. b. Todd should include the $500 in 2019 gross income in accordance with the tax benefit rule. c. Todd should add the $500 to his sales proceeds from the building. d. Todd should include the $500 in 2019 gross income in accordance with the claim of right doctrine. e. None of these.

ANSWER: b RATIONALE: As a cash basis taxpayer, Todd can deduct the one-year prepayment for insurance in the year it was paid, 2018. Because he deducted $1,200 and his net cost was only $700 ($1,200 - $500), Todd should include the $500 refund in gross income for 2019 under the tax benefit rule

Tommy, a senior at State College, receives free room and board as full compensation for working as a resident advisor at the university dormitory. The regular housing contract is $2,000 a year in total, $1,200 for lodging and $800 for meals in the dormitory. Tommy had the option of receiving the meals or $800 in cash. Tommy accepted the meals. What must Tommy include in gross income from working as a resident advisor? a. All items can be excluded from gross income as a scholarship. b. The meals must be included in gross income. c. The meals may be excluded because he did not receive cash. d. The lodging must be included in gross income because it was compensation for services. e. None of these

ANSWER: b RATIONALE: Because Tommy had the option to receive meals or cash, the meals were not provided for the convenience of the employer and, therefore, the value of the meals must be included in gross income. The housing on the employer's premises is received as a condition of employment and, therefore, can be excluded from gross income. Treasury Regulation section 1.119-1(e).

. Benita incurred a business expense on December 10, 2018, which she charged on her bank credit card. She paid the credit card statement which included the charge on January 5, 2019. Which of the following is correct? a. If Benita is a cash method taxpayer, she cannot deduct the expense until 2019. b. If Benita is an accrual method taxpayer, she can deduct the expense in 2018. c. If Benita uses the accrual method, she can choose to deduct the expense in either 2018 or 2019. d. Only b. and c. are correct. e. a., b., and c. are correct.

ANSWER: b RATIONALE: Choice a. is incorrect because charging the expense on a bank credit card is treated as a constructive payment. Thus, as a cash method taxpayer, she can deduct the expense in 2018. If Benita uses the accrual method, she deducts the expense in 2018. In any event, she does not merely choose the year in which to deduct the expense (choice c.).

The de minimis fringe benefit: a. Exclusion applies only to property received by the employee. b. Can be provided on a discriminatory basis. c. Exclusion is limited to $250 per year. d. Exclusion applies to employee discounts. e. None of these

ANSWER: b RATIONALE: Choice a. is incorrect because the exclusion applies to property and services. Choice c. is incorrect because there is no specific dollar amount limitation. Choice d. is incorrect because there are specific rules provided for employee discounts.

In which, if any, of the following situations will the kiddie tax not apply? a. The child is married but does not file a joint return. b. The child has unearned income of $2,100 or less. c. The child has unearned income that exceeds more than half of his (or her) support. d. The child is under age 24 and a full-time student. e. None of these.

ANSWER: b RATIONALE: Choice c. would be appropriate if the reference was to earned (not unearned) income. Choice a. would be correct if a joint return is filed. As to choice d., the kiddie tax could apply if the child was under age 24.

Heather is a full-time employee of the Drake Company and participates in the company's flexible spending plan that is available to all employees. Which of the following is correct? a. Heather reduced her salary by $1,200, actually spent $1,500, and received only $1,200 as reimbursement for her medical expenses. Heather's gross income will be reduced by $1,500. b. Heather reduced her salary by $1,200, and received only $900 as reimbursement for her actual medical expenses. She is not refunded the $300 remaining balance, but her gross income is reduced by $1,200. c. Heather reduced her salary by $1,200, and received only $800 as reimbursement for her medical expenses. She is not refunded the $400. Her gross income is reduced by $800. d. Heather reduced her salary by $1,200, and received only $900 as reimbursement for her medical expenses. She forfeits the $300. Her gross income is reduced by $300. e. None of these.

ANSWER: b RATIONALE: Choices a. and c. are incorrect because to qualify for exclusion treatment, the employee must either use the funds for the designated purpose or forfeit any unused portion of the salary reduction. Choice d. is incorrect because Heather's gross income is reduced by the full $1,200 salary reduction

When using the automatic mileage method, which, if any, of the following expenses also can be claimed? a. Engine tune-up. b. Parking. c. Interest on automobile loan. d. MACRS depreciation. e. None of these.

ANSWER: b RATIONALE: Choices a., c., and d. would be allowed under the actual cost method.

Perry is in the 32% tax bracket. During 2018, he had the following capital asset transactions: Gain from the sale of a stamp collection (held for 10 years) $30,000 Gain from the sale of an investment in land (held for 4 years) 10,000 Gain from the sale of stock investment (held for 8 months) 4,000 Perry's tax consequences from these gains are as follows: a. (15% × $30,000) + (32% × $4,000). b. (15% × $10,000) + (28% × $30,000) + (32% × $4,000). c. (0% × $10,000) + (28% × $30,000) + (32% × $4,000). d. (15% × $40,000) + (32% × $4,000). e. None of these.

ANSWER: b RATIONALE: Collectibles are taxed at a maximum of 28%, while long-term capital gains are subject to a rate of 15% for taxpayers in the 32% tax bracket. Short-term capital gains are treated the same as ordinary income.

Tom operates an illegal drug-running operation and incurred the following expenses: Salaries $ 75,000 Illegal kickbacks 20,000 Bribes to border guards 25,000 Cost of goods sold 160,000 Rent 8,000 Interest 10,000 Insurance on furniture and fixtures 6,000 Utilities and telephone 20,000 Which of the above amounts reduces his taxable income? a. $0. b. $160,000. c. $279,000. d. $324,000. e. None of the above.

ANSWER: b RATIONALE: Cost of goods sold of $160,000 is treated as a negative item in calculating gross income rather than as a deduction. For a drug dealer, all deductions are disallowed.

Ayla, age 17, is claimed by her parents as a dependent. During 2018, she had interest income from a bank savings account of $2,000 and income from a part-time job of $4,200. Ayla's taxable income is: a. $4,200 - $4,550 = $0. b. $6,200 - $5,700 = $500. c. $6,200 - $4,550 = $1,650. d. $6,200 - $1,000 = $5,200. e. None of these.

ANSWER: c RATIONALE: Ayla's standard deduction is $4,200 (earned income) + $350 = $4,550. Thus, her taxable income is $1,650 ($6,200 - $4,550).

On November 1, 2018, Bob, a cash basis taxpayer, gave Dave common stock. On October 30, 2018, the corporation had declared the dividend payable to shareholders of record as of November 22, 2018. The dividend was paid on December 15, 2018. The corporation has paid the $1,200 dividend once each year for the past ten years, during which Bob owned the stock. When Dave collected the dividend on December 15, 2018: a. Bob must include $1,000 (10/12 x $1,200) of the dividend in his gross income. b. Bob must include all of the dividend in his gross income. c. Dave must include all of the dividend in his gross income. d. Dave should treat the $1,200 as a recovery of capital. e. None of these is correct.

ANSWER: b RATIONALE: Dave received the stock as a gift. According to the Tax Court, when the donor makes the gift after the declaration but prior to the record date, the dividend is included in the gross income of the donor.

Debbie is age 67 and unmarried and her only sources of income are $200,000 in taxable interest and $20,000 of Social Security benefits. Debbie's adjusted gross income for the year is: a. $220,000. b. $217,000. c. $203,000. d. $200,000. e. None of these.

ANSWER: b RATIONALE: Debbie must include 85% of her Social Security benefits in gross income. Therefore, her adjusted gross income is $217,000 [$200,000 + (85% × $20,000)].

Detroit Corporation sued Chicago Corporation for intentional damage to Detroit's goodwill. Detroit had created its goodwill through providing high-quality services to its customers. Thus, no basis for the goodwill appeared on Detroit's balance sheet. The suit was settled and Detroit received $1,500,000 for the damages to its goodwill. a. The $1,500,000 is not taxable because it represents a recovery of capital. b. The $1,500,000 is taxable because Detroit has no basis in the goodwill. c. The $1,500,000 is not taxable because Detroit did nothing to earn the money. d. The $1,500,000 is not taxable because Detroit settled the case. e. None of these.

ANSWER: b RATIONALE: Detroit had no basis in the goodwill; therefore, the entire amount received is included in gross income

Barry, a solvent individual but a recovering alcoholic, embezzled $6,000 from his employer. In the same year that he embezzled the funds, his employer discovered the theft. His employer did not fire him and told him he did not have to repay the $6,000 if he would attend Alcoholics Anonymous. Barry met the conditions and his employer canceled the debt. a. Barry did not realize any income because his employer made a gift to him. b. Barry must include $6,000 in gross income from discharge of indebtedness. c. Barry must include $6,000 in gross income under the tax benefit rule. d. Barry may exclude the $6,000 from gross income because the debt never existed. e. None of these

ANSWER: b RATIONALE: Even if the employer had intended that a gift be made, § 102(c) prohibits exclusion treatment. Barry realized a $6,000 increase in his net worth as a result of the theft and the subsequent cancellation of the debt.

In the case of interest income from state and Federal bonds: a. Interest on United States government bonds received by a state resident can be subject to that state's income tax. b. Interest on United States government bonds is subject to Federal income tax. c. Interest on bonds issued by State A received by a resident of State B cannot be subject to income tax in State B. d. All of these are correct. e. None of these are correct.

ANSWER: b RATIONALE: Federal statute prohibits the states from taxing interest on United States government bonds (just as state bond interest is not subject to Federal income tax) (Choice a.). The Federal government does tax its own bonds (Choice b.), and there is no statutory or Constitutional limitation on the states taxing each other's obligations (Choice c.).

Zeke made the following donations to qualified charitable organizations during the year: Basis Fair Market Value Used clothing (all acquired more than a year ago) of taxpayer and his family $ 1,350 $ 375 Stock in ABC, Inc., held as an investment for fifteen months 12,000 10,875 Stock in MNO, Inc., held as an investment for eleven months 15,000 18,000 Real estate held as an investment for two years 15,000 30,000 The used clothing was donated to the Salvation Army; the other items of property were donated to Eastern State University. Both are qualified charitable organizations. Disregarding percentage limitations, Zeke's charitable contribution deduction for the year is: a. $43,350. b. $56,250. c. $59,250. d. $60,375. e. None of the above.

ANSWER: b RATIONALE: For the used clothing and the ABC stock, fair market value controls in determining the amount of the deduction. The ABC stock was held long term, but it was not appreciated property. The MNO stock would not yield a long-term capital gain if sold because of the holding period. Consequently, it is ordinary income property for charitable contribution purposes and the appreciation cannot be claimed. The real estate meets the definition of capital gain property. Thus, $56,250 ($375 + $10,875 + $15,000 + $30,000) is the amount qualifying as a charitable contribution.

Your friend Scotty informs you that he received a "tax-free" reimbursement in 2018 of some medical expenses he paid in 2017. Which of the following statements best explains why Scotty is not required to report the reimbursement in gross income? a. Scotty itemized deductions in 2017. b. Scotty did not itemize deductions in 2017. c. Scotty itemized deductions in 2018. d. Scotty did not itemize deductions in 2018. e. Scotty itemized deductions in 2018 but not in 2017.

ANSWER: b RATIONALE: If Scotty did not itemize in 2017, he can exclude the reimbursement from gross income in 2018. If Scotty itemized deductions in 2017, he must report the reimbursement as gross income in 2018 to the extent he received a tax benefit from deducting medical expenses in 2017. Whether he itemized in 2018 will have no impact on the treatment of the reimbursement.

Fred and Lucy are married, ages 33 and 32, and together have AGI of $120,000 in 2018. They have four dependents and file a joint return. They pay $5,000 for a high deductible health insurance policy and contribute $2,600 to a qualified Health Savings Account. During the year, they paid the following amounts for medical care: $9,200 in doctor and dentist bills and hospital expenses, and $3,000 for prescribed medicine and drugs. In October 2018, they received an insurance reimbursement of $4,400 for the hospitalization. They expect to receive an additional reimbursement of $1,000 in January 2019. Determine the maximum itemized deduction allowable for medical expenses in 2018. a. $800 b. $3,800 c. $9,200 d. $12,800 e. None of the above

ANSWER: b RATIONALE: Fred and Lucy can claim an itemized medical expense deduction for the current year of $3,800, determined as follows: Physician bills, dentist bills, and hospital expenses $ 9,200 Less: Reimbursement (4,400) Unreimbursed expenses $ 4,800 Health insurance premiums 5,000 Prescribed medicines and drugs 3,000 Total medical expenses $12,800 Less: 7.5% of $120,000 (AGI) (9,000) Deductible medical expenses $ 3,800 The contribution of $2,600 to the HSA is a deduction for AGI, and is not included in the medical expense calculation.

Gordon, an employee, is provided group term life insurance coverage equal to twice his annual salary of $125,000 per year. According to the IRS Uniform Premium Table (based on Gordon's age), the amount is $12 per year for $1,000 of protection. The cost of an individual policy would be $15 per year for $1,000 of protection. Since Gordon paid nothing towards the cost of the $250,000 protection, Gordon must include in his 2018 gross income which of the following amounts? a. $1,350. b. $2,400. c. $3,000. d. $3,750. e. None of these.

ANSWER: b RATIONALE: Gordon must include in gross income the Uniform Premium Table amount for $200,000 ($250,000 coverage less the $50,000 exclusion): 200 × $12 = $2,400.

Fran is a CPA who has a small tax practice in addition to working as the controller for a local manufacturing business. Fran runs her tax practice out of a 150 square foot office in her home where she meets clients and works on their tax returns and researches their tax issues. She meets the exclusive use test for this space. The gross income from her tax practice amounts to $7,500 for the year. Business expenses amount to $1,000. Based on square footage, $4,000 of Fran's mortgage interest and real estate taxes are allocable to the home office. The allocable portion of maintenance, utilities, and depreciation is $4,500. Assuming no other expenses related to the business were incurred, what amount of the maintenance, utilities, and depreciation is deductible by Fran? a. $0. b. $2,500. c. $3,500. d. $4,500. e. None of the above

ANSWER: b RATIONALE: Home office expenses cannot generate a loss, and expenses are deducted in the following order: (1) business expenses, (2) interest and taxes, (3) operating expenses of home, and (4) depreciation. Net income before items (3) and (4) amounts to $2,500. As a result, the maximum Fran can only deduct $2,500 of the maintenance, utilities, and depreciation. The balance ($2,000; $4,500 − $2,500) carries over to the following year

Byron owned stock in Blossom Corporation that he donated to a museum (a qualified charitable organization) on June 8 this year. What is the amount of Byron's deduction assuming that he had purchased the stock for $10,500 last year on August 7, and the stock had a fair market value of $13,800 when he made the donation? a. $3,300 b. $10,500 c. $12,150 d. $13,800 e. None of the above

ANSWER: b RATIONALE: If ordinary income property is contributed to a qualified charitable organization, the deduction is equal to the fair market value of the property less the amount of ordinary income that would have been reported if the property were sold. In most instances, the deduction is limited to basis of the property to the donor. Since he had not held the property long enough to meet the long-term capital gain requirement (i.e., not more than one year), Byron would have recognized a short-term capital gain of $3,300 if he had sold the property. Since short-term capital gain property is treated as ordinary income property for charitable contribution purposes, Byron's charitable contribution deduction is limited to the property's basis of $10,500 ($13,800 - $3,300).

Hannah makes the following charitable donations in the current year: Basis Fair Market Value Inventory held for resale in Hannah's business (a sole proprietorship) $ 8,000 $ 7,200 Stock in HBM, Inc., held as an investment (acquired four years ago) 16,000 40,000 Baseball card collection held as an investment (acquired six years ago) 4,000 20,000 The HBM stock and the inventory were given to Hannah's church, and the baseball card collection was given to the United Way. Both donees promptly sold the property for the stated fair market value. Disregarding percentage limitations, Hannah's current charitable contribution deduction is: a. $28,000. b. $51,200. c. $52,000. d. $67,200. e. None of the above

ANSWER: b RATIONALE: Inventory is ordinary income property, but the fair market value ($7,200) must be used if lower than the basis ($8,000). Stock is intangible property and is not subject to the tangible personalty rules. Since a sale of the HBM stock would have yielded a long-term capital gain, the full fair market value qualifies for the deduction ($40,000). The baseball card collection comes under the tangible personalty exception, and the basis ($4,000) must be used. Thus, $51,200 may be deducted as a charitable contribution ($7,200 + $40,000 + $4,000 = $51,200).

Which of the following items would be an itemized deduction on Schedule A of Form 1040? a. Professional dues paid by an accountant (employed by Ford Motor Co.) to the National Association of Accountants. b. Gambling losses to the extent of gambling winnings. c. Job hunting costs. d. Subscription to the Wall Street Journal. e. None of the above.

ANSWER: b RATIONALE: Items a., c., and d. are miscellaneous deductions previously subject to the 2%-ofAGI floor and no longer deductible after the TCJA of 2017.

Marsha is single, had gross income of $50,000, and incurred the following expenses: Charitable contribution $2,000 Taxes and interest on home 7,000 Legal fees incurred in a tax dispute 1,000 Medical expenses 3,000 Penalty on early withdrawal of savings 250 Her AGI is: a. $39,750. b. $49,750. c. $40,000. d. $39,750. e. None of the above.

ANSWER: b RATIONALE: Marsha's AGI is calculated as follows: Gross income $50,000 Deductions for AGI: Penalty on early withdrawal of savings (250) AGI $49,750

Petal, Inc. is an accrual basis taxpayer. Petal uses the aging approach to calculate the reserve for bad debts. During 2018, the following occur associated with bad debts. Credit sales $400,000 Collections on credit sales 250,000 Amount added to the reserve 10,000 Beginning balance in the reserve -0- Identifiable bad debts during 2018 12,000 The amount of the deduction for bad debt expense for Petal for 2018 is: a. $10,000. b. $12,000. c. $22,000. d. $140,000. e. None of the above.

ANSWER: b RATIONALE: Only the specific charge-off method can be used. Reserves for estimated expenses are not allowed for tax purposes because the economic performance test cannot be satisfied.

Quinn, who is single and lives alone, is physically handicapped as a result of a diving accident. In order to live independently, he modifies his personal residence at a cost of $30,000. The modifications included widening halls and doorways for a wheelchair, installing support bars in the bathroom and kitchen, installing a stairway lift, and rewiring so he could reach electrical outlets and appliances. Quinn pays $200 for an appraisal that places the value of the residence at $129,000 before the improvements and $140,000 after. As a result of the operation of the stairway lift, Quinn experienced an increase of $680 in his utility bills for the current year. Disregarding the AGI floor for medical expenses, how much of the above expenditures qualify as medical expense deductions? a. $11,680 b. $30,680 c. $30,880 d. $34,880 e. None of the above

ANSWER: b RATIONALE: Quinn, who is physically handicapped, modified his residence so he could live independently. Therefore, the $30,000 cost of the improvements is not reduced by the $11,000 ($140,000 - $129,000) increase in the value of the house. The additional operating expenses of $680 are deductible as a medical expense. The $200 appraisal fee is not deductible as a medical expense. Rather, it is a miscellaneous itemized deduction subject to the 2%-of-AGI floor. However, the deduction for miscellaneous itemized deductions has been suspended from 2018 through 2025. Therefore, $30,680 ($30,000 + $680) qualifies as medical expenses.

The Blue Utilities Company paid Sue $2,000 for the right to lay an underground electric cable across her property anytime in the future. a. Sue must recognize $2,000 gross income in the current year if the company did not install the cable during the year. b. Sue is not required to recognize gross income from the receipt of the funds, but she must reduce her cost basis in the land by $2,000. c. Sue must recognize $2,000 gross income in the current year regardless of whether the company installed the cable during the year. d. Sue must recognize $2,000 gross income in the current year, and when the cable is installed, she must reduce her cost basis in the land by $2,000. e. None of these.

ANSWER: b RATIONALE: Rationale: The $2,000 payment results in a reduction of Sue's basis in the land but is not income recognized.

Sarah, a majority shareholder in Teal, Inc., made a $200,000 interest-free loan to the corporation. Sarah is not an employee of the corporation. a. Sarah must recognize imputed interest expense and the corporation must recognize imputed interest income. b. Sarah must recognize imputed interest income and the corporation must recognize imputed interest expense. c. Sarah must recognize imputed dividend income and the corporation may recognize imputed interest expense. d. Neither Sarah's nor the corporation's gross income is affected by the loans because no interest was charged. e. None of these.

ANSWER: b RATIONALE: Sarah, as the lender, has imputed interest income. The corporation, as the borrower, has imputed interest expense.

Rachel is single and has a college degree in finance. She is employed as a loan officer at a bank; her yearly AGI approximates $50,000. During the year, she enrolled in a weekend MBA program and incurred the following nonreimbursed expenses: $4,100 (tuition), $300 (books), $200 (other school supplies), and $200 (transportation to and from campus). As to the MBA program, Rachel has a: a. Deduction for and deduction from AGI of $0. b. Deduction for AGI of $4,000 and deduction from AGI of $0. c. Deduction for AGI of $4,000 and deduction from AGI of $800. d. Deduction for AGI of $4,100 and deduction from AGI of $700. e. None of these.

ANSWER: b RATIONALE: Section 222 allows up to $4,000 for qualified tuition and related expenses. As Rachel spent $4,100, she can claim only $4,000 as a deduction for AGI. The remaining expenses of $800 ($100 + $300 + $200 + $200) are miscellaneous itemized deductions (deductions from AGI), and are not deductible from 2018 through 2025.

Emily is in the 35% marginal tax bracket. She can purchase a York County school bond yielding 3.5% interest and the interest is not subject to a 5% state tax. But she is interested in earning a higher return for comparable risk. Which of the following is correct: a. If she buys a corporate bond that pays 6% interest, her after-tax rate of return will be less than if she purchased the York County school bond. b. If she buys a U.S. government bond paying 5%, her after-tax rate of return will be less than if she purchased the York County school bond. c. If she buys a common stock paying a 4% dividend, her after-tax rate of return will be higher than if she purchased the York County school bond. d. All of these are correct. e. None of these are correct.

ANSWER: b RATIONALE: See the table below: Before Tax Tax @ .40; .35*; .20** After-tax York County Bond a. 6.00% 2.400% 3.600% 3.5% b. 5.00% 1.750% 3.250% 3.5% c. 4.00% .8% 3.200% 3.5% *The state does not tax the Federal bonds. **The dividend is taxed at 15% Federal and 5% state.

Iris, a calendar year cash basis taxpayer, owns and operates several TV rental outlets in Florida, and wants to expand to other states. During 2018, she spends $14,000 to investigate TV rental stores in South Carolina and $9,000 to investigate TV rental stores in Georgia. She acquires the South Carolina operations, but not the outlets in Georgia. As to these expenses, Iris should: a. Capitalize $14,000 and not deduct $9,000. b. Expense $23,000 for 2018. c. Expense $9,000 for 2018 and capitalize $14,000. d. Capitalize $23,000. e. None of the above

ANSWER: b RATIONALE: Since Iris owns and operates TV rental outlets, all of the investigation expenses can be deducted.

Paul, a calendar year single taxpayer, has the following information for 2018: AGI $175,000 State income taxes 13,500 State sales tax 3,000 Real estate taxes 18,900 Gambling losses (gambling gains were $12,000) 6,800 Paul's allowable itemized deductions for 2018 are: a. $10,000. b. $16,800. c. $39,200. d. $42,200. e. None of the above

ANSWER: b RATIONALE: State taxes are limited to $10,000. The itemized deductions allowed are $16,800 ($10,000 + $6,800).

Which, if any, of the following is subject to an overall limitation on meals in 2018? a. Meals provided to employees during a business meeting. b. Meals provided at cost to employees by a cafeteria funded by the employer. c. Fourth of July company picnic for employees. d. Meals provided to employees during a training event or retreat at an off-site location. e. All of these.

ANSWER: b RATIONALE: Subsidized eating facilities are subject to a 50% limitation beginning in 2018 (and no deductions will be allowed beginning in 2026).

Terry and Jim are both involved in operating illegal businesses. Terry operates a gambling business and Jim operates a drug running business. Both businesses have gross revenues of $500,000. The businesses incur the following expenses. Terry Jim Employee salaries $200,000 $200,000 Bribes to police 25,000 25,000 Rent and utilities 50,000 50,000 Cost of goods sold -0- 125,000 Which of the following statements is correct? a. Neither Terry nor Jim can deduct any of the above items in calculating the business profit. b. Terry should report profit from his business of $250,000. c. Jim should report profit from his business of $500,000. d. Jim should report profit from his business of $250,000. e. None of the above.

ANSWER: b RATIONALE: Terry and Jim should report net profit from their businesses as follows: Terry Jim Gross revenues $500,000 $500,000 Less: Cost of goods sold (-0-) (125,000) Gross income $500,000 $375,000 Less: Expenses Employee salaries (200,000) (-0-) Rent and utilities (50,000) (-0-) Bribes to police (-0-) (-0-) Net profit $250,000 $375,000 For Terry, the bribes to the police of $25,000 cannot be deducted. None of Jim's expenses can be deducted. However, the cost of goods sold is viewed as a negative item in calculating gross income (i.e., gross income = gross profit) rather than as a deduction.

Ron, age 19, is a full-time graduate student at City University. During 2018, he received the following payments: Cash award for being the outstanding resident adviser $ 1,500 Resident adviser housing 2,500 State scholarship for ten months (tuition and books) 6,000 State scholarship (meals allowance) 2,400 Loan from college financial aid office 3,000 Cash support from parents 2,000 $17,400 Ron served as a resident adviser in a dormitory and, therefore, the university waived the $2,500 charge for the room he occupied. What is Ron's adjusted gross income for 2018? a. $1,500. b. $3,900. c. $9,000. d. $15,400. e. None of these.

ANSWER: b RATIONALE: The $1,500 award for being the outstanding resident adviser is included in Ron's gross income. The $2,400 meal allowance must also be included in Ron's gross income. The $2,500 waiver for housing is not included in Ron's gross income because the housing is provided for the convenience of his employer. The $6,000 scholarship for tuition and books is excluded from his gross income. The loan of $3,000 is not income.

The taxpayer is a Ph.D. student in accounting at City University. The student is paid $1,500 per month for teaching two classes. The total amount received for the year is $13,500. a. The $13,500 is excludible if the money is used to pay for tuition and books. b. The $13,500 is taxable compensation. c. The $13,500 is considered a scholarship and, therefore, is excluded. d. The $13,500 is excluded because the total amount received for the year is less than her standard deduction and personal exemption. e. None of these.

ANSWER: b RATIONALE: The $13,500 represents compensation for services rendered and must be included in the student's gross income.

James, a cash basis taxpayer, received the following compensation and fringe benefits in the current year: Salary $66,000 Disability income protection premiums 3,000 Long-term care insurance premiums 4,000 His actual salary was $72,000. He received only $66,000 because his salary was garnished and the employer paid $6,000 on James's credit card debt he owed. The wage continuation insurance is available to all employees and pays the employee three-fourths of the regular salary if the employee is sick or disabled. The long-term care insurance is available to all employees and pays $150 per day towards a nursing home or similar facility. What is James's gross income from the above? a. $66,000. b. $72,000. c. $73,000. d. $75,000. e. None of these.

ANSWER: b RATIONALE: The $6,000 paid to the creditor is income to James as it was used to pay his debt. The disability income protection premiums and long-term care insurance premiums are excluded from gross income.

Under the terms of a divorce agreement entered into in 2017, Ron is to pay his former wife Jill $10,000 per month. The payments are to be reduced to $7,000 per month when their 15 year-old child reaches age 18. During the current year, Ron paid $120,000 under the agreement. Assuming all of the other conditions for alimony are satisfied, Ron can deduct from gross income (and Jill must include in gross income) as alimony: a. $120,000. b. $84,000. c. $36,000. d. $0. e. None of these is correct.

ANSWER: b RATIONALE: The amount paid is in part dependent upon the child's age, and therefore is considered child support. The balance of the payments, $7,000 per month, is alimony.

Sandra is single and does a lot of business entertaining at home. Because Arthur, Sandra's 80-year old dependent grandfather who lived with Sandra, needs medical and nursing care, he moved to Twilight Nursing Home. During the year, Sandra made the following payments on behalf of Arthur: Room at Twilight $4,500 Meals for Arthur at Twilight 850 Doctor and nurse fees 700 Cable TV service for Arthur's room 107 Total $6,157 Twilight has medical staff in residence. Disregarding the AGI floor, how much, if any, of these expenses qualify for a medical deduction by Sandra? a. $6,157 b. $6,050 c. $5,200 d. $1,550 e. None of the above

ANSWER: b RATIONALE: The amount that qualifies is $6,050 ($4,500 + $850 + $700). The room and board for Twilight qualifies because the move was motivated by Arthur's need for medical care. The cable fee is a personal expense and cannot be deducted.

Harold bought land from Jewel for $150,000. Harold paid $50,000 cash and gave Jewel an 8% note for $100,000. The note was to be paid over a five-year period. When the balance on the note was $80,000, Jewel began having financial difficulties. To accelerate her cash inflows, Jewel agreed to accept $60,000 cash from Harold in final payment of the note principal. a. Harold must recognize $20,000 ($80,000 - $60,000) of gross income. b. Harold is not required to recognize gross income, but must reduce his cost basis in the land to $130,000. c. Harold is not required to recognize gross income, since he paid the debt before it was due. d. Jewel must recognize gross income of $20,000 ($80,000 - $60,000) from discharge of the debt. e. None of these.

ANSWER: b RATIONALE: The debt reduction of $20,000 is treated as an adjustment to the cost of the land because the debt was owed to the seller of the property.

The taxable portion of Social Security benefits may be affected by: a. The taxpayer's itemized deductions. b. The individual's tax-exempt interest income. c. The number of quarters the individual worked. d. The individual's standard deduction. e. None of these.

ANSWER: b RATIONALE: The formula used to calculate the taxable portion of Social Security requires the taxpayer to aggregate the taxpayer's Social Security benefits and other sources of income, including tax-exempt interest. If this total income exceeds a base amount, as much as 85% of the benefits can be subject to Federal income tax.

Carin, a widow, elected to receive the proceeds of a $150,000 life insurance policy on the life of her deceased husband in 10 installments of $17,500 each. Her husband had paid premiums of $60,000 on the policy. In the first year, Carin collected $17,500 from the insurance company. She must include in gross income: a. $0. b. $2,500. c. $10,000. d. $25,000. e. None of these.

ANSWER: b RATIONALE: The interest element of $25,000 ($175,000 - $150,000) is included in Carin's gross income. The payments are taxed as an annuity, and therefore $2,500 [($25,000/$175,000) × $17,500] is included in Carin's gross income in the first year

For the current year, David has wages of $80,000 and the following property transactions: Stock investment sales— Long-term capital gain $ 9,000 Short-term capital loss (12,000) Loss on sale of camper (purchased 4 years ago and used for family vacations) (2,000) What is David's AGI for the current year? a. $76,000. b. $77,000. c. $78,000. d. $89,000. e. None of these

ANSWER: b RATIONALE: The loss from the sale of the camper is personal and, therefore, is not deductible. Netting the short-term capital loss of $12,000 against the long-term capital gain of $9,000 produces a net short-term capital loss of $3,000. Offsetting the capital loss against ordinary income yields AGI of $77,000 ($80,000 - $3,000)

Barney is a full-time graduate student at State University. He serves as a teaching assistant for which he is paid $700 per month for 9 months and his $5,000 tuition is waived. The university waives tuition for all of its employees. In addition, he receives a $1,500 research grant to pursue his own research and studies. Barney's gross income from the above is: a. $0. b. $6,300. c. $11,300. d. $12,800. e. None of these.

ANSWER: b RATIONALE: The monthly stipend of $700 (for 9 months = $6,300) for teaching is taxable compensation. The research grant of $1,500 is to assist him in his education and is not in exchange for services; therefore, the grant is a nontaxable scholarship. The tuition waiver is excluded as a qualified tuition reduction program

The Green Company, an accrual basis taxpayer, provides business-consulting services. Clients generally pay a retainer at the beginning of a 12-month period. This entitles the client to no more than 40 hours of services. Once the client has received 40 hours of services, Green charges $500 per hour. Green Company allocates the retainer to income based on the number of hours worked on the contract. At the end of the tax year for contracts entered into for the current year, the company had $50,000 of unearned revenues from these contracts. The company also had $10,000 in unearned rent income received this year from excess office space leased to other companies. Based on the above, Green must include in gross income for the subsequent tax year: a. $60,000. b. $50,000. c. $10,000. d. $0. e. None of these.

ANSWER: b RATIONALE: The prepaid income from services not reported in the year received must all be reported in the next year for tax purposes. The prepaid income from rents is not eligible for deferral.

A worker may prefer to be classified as an employee (rather than an independent contractor) for which of the following reasons: a. To claim unreimbursed work-related expenses as a deduction for AGI. b. To avoid the self-employment tax. c. To avoid the overall limitation (50%) on unreimbursed business entertainment expenses. d. To avoid the limitations on unreimbursed work-related expenses. e. None of these

ANSWER: b RATIONALE: The self-employment tax is twice the Social Security equivalent imposed on employees (choice b.). Most unreimbursed employee expenses are deductions from AGI as miscellaneous itemized deductions; these deductions have been susptended from 2018 through 2025 (choice a. and choice d.). Unreimbursed ente

The annual increase in the cash surrender value of a life insurance policy: a. Is taxed according to the original issue discount rules. b. Is not included in gross income because the policy must be surrendered to receive the cash surrender value. c. Reduces the deduction for life insurance expense. d. Is exempt because it is life insurance proceeds. e. None of these.

ANSWER: b RATIONALE: The substantial restrictions on gaining access to the policy, the fact the taxpayer must cancel the policy means that the increase in value is not actually or constructively received.

Pedro's child attends a school operated by the church the family attends. Pedro made a donation of $1,000 to the church in lieu of the normal registration fee of $200. In addition, Pedro paid the regular tuition of $6,000 to the school. Based on this information, what is Pedro's charitable contribution? a. $0 b. $800 c. $1,000 d. $6,800 e. $7,000

ANSWER: b RATIONALE: The taxpayer's donation of $1,000 in lieu of the normal $200 registration fee would be deductible to the extent of $800 [$1,000 - $200 benefit received (the registration fee)]. The tuition of $6,000 is a personal expense that cannot be deducted as a charitable contribution.

Tom, age 48, is advised by his family physician that he needs back surgery to correct a problem from his last back surgery. Since Tom is in a wheel chair, he needs his wife, Jean, to accompany him on his trip to Rochester, Minnesota, for in-patient treatment at the Mayo Clinic, which specializes in this type of surgery. Tom incurred the following costs in 2018: Round-trip airfare ($350 each) $ 700 Jean's hotel in Rochester for four nights ($95 per night) 380 Jean's meals while in Rochester 105 Tom's medical treatment 3,500 Tom's prescription medicine 600 Compute Tom's allowable medical expenses for the trip (before application of the AGI floor). a. $4,000 b. $5,000 c. $5,180 d. $5,285 e. None of the above

ANSWER: b RATIONALE: Tom's medical expense deduction for transportation is $700, and his medical expense deduction for lodging for Jean is $200 ($50 per night per person maximum). Tom is not allowed a deduction for the cost of Jean's meals while in Rochester. His medical treatment and prescriptions are deductible. So his total deduction (before application of the AGI floor) is $5,000 ($700 + $200 + $3,500 + $600)

For purposes of determining gross income, which of the following is true? a. A mechanic completed repairs on an automobile during the year and collects money from the customer. The customer was not satisfied with the repairs and sued the mechanic for a refund. The mechanic can defer recognition of the income until the suit has been settled. b. A taxpayer who finds a wallet full of money is required to recognize income even though someone may eventually ask for the return of the money. c. Embezzlement proceeds are not included in the embezzler's gross income because the embezzler has an obligation to repay the owner. d. All of these are false. e. All of these are true

ANSWER: b RATIONALE: Under the claim of right doctrine, the person who finds property, and has free and unrestricted use of the property, has realized an increase in wealth and must include the found property in his or her gross income.

During the year, Walt (self-employed) travels from Seattle to Tokyo (Japan) on business. His time was spent as follows: 2 days travel (one day each way), 2 days business, and 2 days personal. His expenses for the trip were as follows (meals and lodging reflect only the business portion): Air fare $3,000 Lodging 2,000 Meals 1,000 Presuming no reimbursement, Walt's deductible expenses are: a. $3,500. b. $4,500. c. $5,500. d. $6,000. e. None of these.

ANSWER: c RATIONALE: $3,000 + $2,000 + (50% × $1,000) = $5,500 . Since the 7-days-or-less exception applies, the full airfare ($3,000) is allowed.

During the year, John (a self-employed management consultant) went from Milwaukee to Alaska on business. Preceding a five-day business meeting, he spent four days vacationing at the beach. Excluding the vacation costs, his expenses for the trip are: Air fare $3,200 Lodging 900 Meals 800 Entertainment 600 Presuming no reimbursement, deductible expenses are: a. $3,200. b. $3,900. c. $4,500. d. $5,500. e. None of these.

ANSWER: c RATIONALE: $3,200 + $900 + $400 [50% x $800] = $4,500. No allocation is required for domesti

Jana has $225,000 of earned income in 2018. Calculate the amount she can contribute to a SEP. a. $22,500 b. $24,500 c. $55,000 d. $56,250 e. None of the above

ANSWER: c RATIONALE: 25% × $225,000 statutory limit = $56,250, but limited to $55,000 (in 2018).

With respect to income from services, which of the following is true? a. The income is always amortized over the period the services will be rendered by an accrual basis taxpayer. b. A cash basis taxpayer can spread the income from a 24-month service contract over the contract period. c. If an accrual basis taxpayer sells a 36-month service contract on July 1, 2018 for $3,600, the taxpayer's 2018 gross income from the contract is $600. d. If an accrual basis taxpayer sells a 24-month service contract on July 1, 2018, one-half (12/24) the income is recognized in 2019. e. None of these.

ANSWER: c RATIONALE: Answer b. is incorrect because the special method for deferral of prepaid income does not apply to cash basis taxpayers. Answers a. and d. are incorrect because they are not in accordance with § 451(c) (previously Revenue Procedure 2004-34).

Phillip, age 66, developed hip problems and was unable to climb the stairs to reach his second-floor bedroom. His physician advised him to add a first-floor bedroom to his home. The cost of constructing the room was $32,000. The increase in the value of the residence as a result of the room addition was determined to be $17,000. In addition, Phillip paid the contractor $5,500 to construct an entrance ramp to his home and $8,500 to widen the hallways to accommodate his wheelchair. Phillip's AGI for 2018 was $75,000. How much of these expenditures can Phillip deduct as a medical expense in 2018? a. $14,000 b. $15,000 c. $23,375 d. $29,000 e. None of the above

ANSWER: c RATIONALE: A capital improvement that ordinarily would not have a medical purpose qualifies as a medical expense if it is directly related to prescribed medical care and is deductible to the extent that the expenditure exceeds the increase in value of the related property. Examples of such improvements include dust elimination systems, elevators, and vans specially designed for wheelchair-bound taxpayers. Phillip's medical expense related to the room addition is $15,000 ($32,000 - $17,000). The full cost of home-related capital expenditures incurred to enable a physically handicapped individual to live independently and productively qualifies as a medical expense. Qualifying costs include expenditures for constructing entrance and exit ramps to the residence, widening hallways and doorways to accommodate wheelchairs, installing support bars and railings in bathrooms and other rooms, and adjusting electrical outlets and fixtures. These expenditures are subject to the AGI floor only, and the increase in the home's value is deemed to be zero. Phillip's medical expense related to the ramp and hallways is $14,000 ($5,500 + $8,500). So Phillip's medical expense deduction is as follows: Qualifying medical expenses ($15,000 + $14,000) $29,000 Less: 7.5% × $75,000 (AGI) (5,625) Deductible medical expenses $23,375

Which of the following would constitute an employer-employee relationship? a. A plumber who comes to your home to fix a leaking faucet. b. A CPA who prepares a client's tax return. c. A physician who hires a nurse to help her with patient screening and preliminary tests in the office. d. A gardener who takes care of individual lawns for a monthly fee. e. None of the above would constitute an employer-employee relationship.

ANSWER: c RATIONALE: A nurse hired by a doctor would be considered an employee

Al is single, age 60, and has gross income of $140,000. His deductible expenses are as follows: Alimony $20,000 Charitable contributions 4,000 Contribution to a traditional IRA 5,500 Expenses paid on rental property 7,500 Interest on home mortgage and property taxes on personal residence 7,200 State income tax 2,200 What is Al's AGI? a. $94,100. b. $103,000. c. $107,000. d. $127,000. e. None of the above.

ANSWER: c RATIONALE: Al's AGI is calculated as follows: Gross income $140,000 Deductions for AGI: Alimony $20,000 IRA 5,500 Expenses on rental property 7,500 (33,000) AGI $107,000

Harry and Wanda were married in Texas, a community property state, but moved to Virginia, a common law state. The calculation of their income on a joint return: a. Will increase as a result of changing their state of residence. b. Will decrease as a result of changing their state of residence. c. Will not change as a result of changing their state of residence. d. Will not be permitted. e. None of these.

ANSWER: c RATIONALE: All of their income, regardless of whether they live in a common law state or community property state, must be included on their joint return.

Which of the following must be capitalized by a business? a. Replacement of a windshield of a business truck which was broken in an accident. b. Repair of a roof of a building used in business. c. Amount paid for a covenant not to compete. d. Only b. and c. must be capitalized. e. a., b., and c. can be expensed rather than capitalized

ANSWER: c RATIONALE: All of these expenses, except for the covenant, can be deducted in the current tax year. The amortization period for the covenant is 15 years.

On January 2, 2018, Fran acquires a business from Chuck. Among the assets purchased are the following intangibles: patent with a 7-year remaining life, a covenant not to compete for 10 years, and goodwill. Of the purchase price, $140,000 was paid for the patent and $60,000 for the covenant. The amount of the excess of the purchase price over the identifiable assets was $100,000. What is the amount of the amortization deduction for 2018? a. $10,667. b. $16,000. c. $20,000. d. $32,667. e. None of the above

ANSWER: c RATIONALE: All of these intangibles are § 197 intangibles and are amortized over a 15-year statutory period. Patent $140,000 ÷ 15 = $ 9,333 Covenant $60,000 ÷ 15 = 4,000 Goodwill $100,000 ÷ 15 = 6,667 $20,000

Jeremy is married to Amy, who abandoned him in 2017. He has not seen or communicated with her since April of that year. He maintains a household in which their son, Evan, lives. Evan is age 25 and earns over $6,000 each year. For tax year 2018, Jeremy's filing status is: a. Married, filing jointly. b. Head of household. c. Married, filing separately. d. Surviving spouse. e. Single.

ANSWER: c RATIONALE: Because Jeremy is still treated as being married, his only option is married, filing separately (choice c.). Jeremy cannot file jointly without Amy's consent (choice a.). He is not an abandoned spouse since Evan is not a dependent child. Evan cannot be claimed as a qualifying child (age test) and is not a qualifying relative (gross income test).

For an activity classified as a hobby, the expenses are categorized as follows: (1) Amounts that affect adjusted basis and would be deductible under other Code sections if the activity had been engaged in for profit (e.g., depreciation, amortization, and depletion). (2) Amounts deductible under other Code sections without regard to the nature of the activity, such as property taxes and home mortgage interest. (3) Amounts deductible under other Code sections if the activity had been engaged in for profit, but only if those amounts do not affect adjusted basis (e.g., maintenance, utilities, and supplies). For tax years before 2018, if these expenses exceed the gross income from the activity and are thus limited, the sequence in which they are deductible is: a. (1), (2), (3). b. (1), (3), (2). c. (2), (3), (1). d. (2), (1), (3). e. (3), (2), (1).

ANSWER: c RATIONALE: Before 2018, the last two categories of deductions were deductible from AGI as miscellaneous itemized deductions (to the extent they exceeded 2 percent of AGI). From 2018 through 2025, miscellaneous itemized deductions are not deductible.

Payments by a cash basis taxpayer of capital expenditures: a. Must be expensed at the time of payment. b. Must be expensed by the end of the first year after the asset is acquired. c. Must be deducted over the actual or statutory life of the asset. d. Can be deducted in the year the taxpayer chooses. e. None of the above.

ANSWER: c RATIONALE: Both cash basis and accrual basis taxpayers are required to recover the cost of capital assets through amortization, depletion, or depreciation over the actual or statutory life of the asset.

Brad, who uses the cash method of accounting, lives in a state that imposes an income tax (including withholding from wages). On April 14, 2018, he files his state return for 2017, paying an additional $600 in state income taxes. During 2018, his withholdings for state income tax purposes amount to $3,550. On April 13, 2019, he files his state return for 2018 claiming a refund of $800. Brad receives the refund on June 3, 2019. If he itemizes deductions, how much may Brad claim as a deduction for state income taxes on his Federal income tax return for calendar year 2018 (filed in April 2019)? a. $3,350 b. $3,550 c. $4,150 d. $5,150 e. None of the above

ANSWER: c RATIONALE: Brad is a cash basis taxpayer. His deduction is limited to the amounts paid in 2018. The $800 refund is reported as income in 2019 under the tax benefit rule. Brad's state income tax deduction for 2018 is determined as follows: Paid April 14, 2018, for 2017 $ 600 Withholdings for 2018 3,550 Total deduction $4,150

Regarding the rules applicable to filing of income tax returns, which, if any, of the following is an incorrect statement: a. Married persons who file joint returns cannot later (after the due date of the return) substitute separate returns. b. Married persons who file separate returns can later (after the due date of the return) substitute a joint return. c. The usual test as to when a taxpayer must file a return is based on the total of the following: personal exemption + basic standard deduction + both additional standard deductions. d. Special filing requirement rules exist for taxpayers who are claimed as dependents of another. e. None of these.

ANSWER: c RATIONALE: Choice c. would be correct if it included only the additional standard deduction for age, as that for blindness is not considered.

Aiden performs services for Lucas. Which, if any, of the following factors indicate that Aiden is an employee, rather than an independent contractor? a. Aiden provides his own support services (e.g., work assistants). b. Aiden obtained his training (i.e., job skills) from his father. c. Aiden is paid based on hours worked. d. Aiden makes his services available to others. e. None of these

ANSWER: c RATIONALE: Choices a., b., and d. reflect independent contractor status.

Kirby is in the 12% tax bracket and had the following capital asset transactions during 2018: Long-term gain from the sale of a coin collection $11,000 Long-term gain from the sale of a land investment 10,000 Short-term gain from the sale of a stock investment 2,000 Kirby's tax consequences from these gains are as follows: a. (5% × $10,000) + (12% × $13,000). b. (12% × $13,000) + (28% × $11,000). c. (0% × $10,000) + (12% × $13,000). d. (12% × $23,000). e. None of these.

ANSWER: c RATIONALE: Collectibles and short-term capital gains are taxed at Kirby's regular 12% tax bracket, while long-term capital gains are subject to a rate of 0%

Edna had an accident while competing in a rodeo. She sustained facial injuries that required cosmetic surgery. While having the surgery done to restore her appearance, she had additional surgery done to reshape her chin, which was not injured in the accident. The surgery to restore her appearance cost $9,000 and the surgery to reshape her chin cost $6,000. How much of Edna's surgical fees will qualify as a deductible medical expense (before application of the 10%-of-AGI floor)? a. $0 b. $6,000 c. $9,000 d. $15,000 e. None of the above

ANSWER: c RATIONALE: Cosmetic surgery is necessary (and therefore deductible) when it ameliorates (1) a deformity arising from a congenital abnormality, (2) a personal injury, or (3) a disfiguring disease. The $9,000 cost incurred in connection with the restorative surgery (required as a result of the accident) is deductible because the surgery was necessary. Amounts paid for the unnecessary cosmetic surgery ($6,000 for reshaping the chin) are not deductible as a medical expense.

Dana, age 31 and unmarried, is an active participant in a qualified retirement plan. Her AGI is $124,000. What amount, if any, may Dana contribute to a Roth IRA in 2018? a. $0. b. $3,225. c. $4,033. d. $5,500. e. None of the above

ANSWER: c RATIONALE: Dana may contribute $4,033 to a Roth IRA in 2018, calculated as follows: $124,000 AGI - $120,000 threshold = $4,000 excess AGI $4,000/$15,000 phaseout range × $5,500 = $1,467 phaseout $5,500 maximum contributions - $1,467 phaseout = $4,033 contribution ceiling

All employees of United Company are covered by a group hospitalization insurance plan, but the employees must pay the premiums ($8,000 for each employee). None of the employees has sufficient medical expenses to deduct the premiums. Instead of giving raises next year, United is considering paying the employee's hospitalization insurance premiums. If the change is made, the employee's after-tax and insurance pay will: a. Decrease by the same amount for all employees. b. Increase more for the lower paid employees (10% and 12% marginal tax bracket). c. Increase more for the higher income (35% marginal tax bracket) employees. d. Increase by the same amount for all employees. e. None of these.

ANSWER: c RATIONALE: Each employee's income, less taxes and insurance, would increase by the cost of insurance times the employee's marginal tax rate. The employees who are in the higher tax brackets will benefit more from the change than the employees in the lower tax brackets.

In 2018 Todd purchased an annuity for $150,000. The annuity is to pay him $2,500 per month for the rest of his life. His life expectancy is 100 months. Which of the following is correct? a. Todd is not required to recognize any income until he has collected 60 payments (60 × $2,500 = $150,000). b. If Todd collects 20 payments and then dies in 2018, Todd's estate should amend his tax returns for 2018 and 2019 and eliminate all of the reported income from the annuity for those years. c. For each $2,500 payment received in the first year, Todd must include $1,000 in gross income. d. For each $2,500 payment received in the first year, Todd must include $1,500 in gross income. e. None of these.

ANSWER: c RATIONALE: Each payment is in part a recovery of capital and in part income. The recovery of capital portion is $1,500 [($150,000 cost/$250,000 expected return) × $2,500 payment]. The balance of the amount received of $1,000 ($2,500 - $1,500) is income.

A company has a medical reimbursement plan for officers that covers all costs that the insurer will not pay. However, for all employees who are not officers, the medical reimbursement plan applies only after the employee has paid $1,000 from his or her own funds. An officer incurred $1,500 in medical expenses and was reimbursed for that amount. An hourly worker also incurred $1,500 in medical expense and was reimbursed $500. a. Both employees must include all benefits received in gross income. b. The officer must include $500 in gross income. c. The officer must include $1,500 in gross income. d. The hourly employee must include $1,000 in gross income. e. None of these.

ANSWER: c RATIONALE: For discriminatory plans, the highly-compensated employees do not qualify for exclusion

Orange Cable TV Company, an accrual basis taxpayer, allows its customers to pay by the year in advance ($600 per year), or two years in advance ($960). In September 2018, the company collected the following amounts applicable to future services: October 2018-September 2020 services (200 two-year contracts) $192,000 October 2018-September 2019 services (200 one-year contracts) 120,000 Total $312,000 As a result of the above, Orange Cable should report as gross income for 2019: a. $54,000. b. $78,000. c. $258,000. d. $312,000. e. None of these

ANSWER: c RATIONALE: For financial reporting purposes, Orange will report 3 months of each of these contracts in 2018: Two-year contracts: 200 x 3 months x $40 $24,000 One-year contracts: 200 x 3 months x $50 30,000 Total revenue for 2018 $54,000 Under the income deferral rule for an accrual method taxpayer (Rev. Proc. 2004-34; replaced by § 451(c) by the TCJA of 2017), the balance of the prepaid income must be reported in the subsequent year ($312,000 - $54,000 = $258,000).

Our tax laws encourage taxpayers to ____ assets that have appreciated in value and ____ assets that have declined in value. a. sell; keep. b. sell; sell. c. keep; sell. d. keep; keep. e. None of these.

ANSWER: c RATIONALE: Gain or loss is not recognized until the asset is sold. When the taxpayer sells assets that have declined in value, the after-tax proceeds are greater than the selling price because the loss reduces taxes otherwise due. Conversely, when an appreciated asset is sold, the after-tax proceeds are less than the selling price because of the tax due on the gain.

A worker may prefer to be treated as an independent contractor (rather than an employee) for which of the following reasons: a. Avoids the overall limitation (50%) as to business meals. b. All of the self-employment tax is deductible for income tax purposes. c. Work-related expenses of an independent contractor are deductible for AGI. d. A Schedule C does not have to be filed. e. None of these

ANSWER: c RATIONALE: Independent contractors are also subject to the overall limitation (50%) (choice a.). All of the self-employment tax is not deductible (choice b.). Work-related expenses of an independent contractor are deductions for AGI. The work-related expenses of an employee are not deductible.(choice c.). Work-related expenses will have to be reported on Schedule C (choice d.).

Under the actual cost method, which, if any, of the following expenses will not be allowed? a. Car registration fees. b. Auto insurance. c. Interest expense on a car loan (taxpayer is an employee). d. Dues to auto clubs. e. All of these will be allowed

ANSWER: c RATIONALE: Interest on a car loan is deductible only by a self-employed taxpayer (not an employee)—choice c

Regarding the tax formula and its relationship to Form 1040, which, if any, of the following statements is correct? a. Most exclusions from gross income are reported on page 2 of Form 1040. b. An "above the line deduction" refers to a deduction from AGI. c. A "page 1 deduction" refers to a deduction for AGI. d. The taxable income (TI) amount appears both at the bottom of page 1 and at the top of page 2 of Form 1040. e. None of these.

ANSWER: c RATIONALE: Most exclusions are not reported on Form 1040, but those that are appear on page 1 —not on page 2 (choice a.). A "page 1 deduction" as well as an "above the line deduction" are deductions for AGI (choices b. and c.). AGI, not TI, is at the bottom of page 1 and the top of page 2 of Form 1040 (choice d.).

Nikeya sells land (adjusted basis of $120,000) to her adult son, Shamed, for its appraised value of $95,000. Which of the following statements is correct? a. Nikeya's recognized loss is $25,000 ($95,000 amount realized - $120,000 adjusted basis). b. Shamed's adjusted basis for the land is $120,000 ($95,000 cost + $25,000 disallowed loss for Nikeya). c. If Shamed subsequently sells the land for $112,000, he has no recognized gain or loss. d. Only a. and b. are correct. e. a., b., and c. are correct.

ANSWER: c RATIONALE: Nikeya's realized loss of $25,000 ($95,000 amount realized - $120,000 adjusted basis) is disallowed because Shamed is a related party. Shamed's adjusted basis for the land is his cost of $95,000. However, when he sells the land for $112,000, his realized gain of $17,000 ($112,000 amount realized - $95,000 adjusted basis) is not recognized because he can offset it against $17,000 of Nikeya's $25,000 disallowed loss in calculating his taxable income.

The tax concept and economic concept of income are in agreement on which of the following: a. The fair rental value of an owner-occupied home should be included in income. b. The increase in value of assets held for the entire year should be included in income for the year. c. Rent income for 2019 collected in 2018 is income for 2018. d. All of these.

ANSWER: c RATIONALE: The realization requirement applies to taxable income, but does not apply to economic income. The prepaid rent income received in 2018 is included in gross income in that year and is an increase in assets in 2018.

During the current year, Ralph made the following contributions to the University of Oregon (a qualified charitable organization): Cash $63,000 Stock in Raptor, Inc. (a publicly traded corporation) 94,500 Ralph acquired the stock in Raptor, Inc., as an investment fourteen months ago at a cost of $42,000. Ralph's AGI for the year is $189,000. What is Ralph's charitable contribution deduction for the current year? a. $56,700 b. $63,000 c. $94,500 Copyright Cengage Learning. Powered by Cognero. Page 51 Chapter 10: Deductions and Losses: Certain Itemized Deductions d. $157,500 e. None of the above

ANSWER: c RATIONALE: No reduction for the appreciation on the Raptor, Inc., stock is necessary because, if sold, it would yield a long-term capital gain. Thus, Ralph's potential charitable deduction for the year is $157,500 [$63,000 (cash) + $94,500 (fair market value of Raptor, Inc., stock)], but his allowable contribution deduction for the year is limited to $94,500 (50% of $189,000 AGI). Although the 30%-of-AGI limitation applies to capital gain property, which would result in a current deduction for the Raptor, Inc., stock of $56,700 (30% of $189,000 AGI), the overall 50%-of-AGI limitation applies to limit the total current deduction to $94,500. When the contributions for the tax year involve both 50% property (the cash of $63,000 in this case) and 30% property (the Raptor, Inc., stock), the allowable deduction comes first from the 50% property. Therefore, Ralph's allowable current deduction of $94,500 consists of: Cash $63,000 Raptor, Inc., stock [overall limitation of $94,500 (50% of $189,000 AGI) - $63,000 (cash)] 31,500 $94,500 The unused portion of the Raptor, Inc., stock contribution of $63,000 [$94,500 (fair market value) - $31,500 (portion used currently)] may be carried over for five years. The carryover continues to be classified as 30% property in the carryover years. If Ralph plans his charitable deductions wisely, sooner or later he will be able to deduct the full $94,500 fair market value of the stock.

During the current year, Khalid was in an automobile accident and suffered physical injuries. The accident was caused by Rashad's negligence. Khalid threatened to file a lawsuit against Amber Trucking Company, Rashad's employer, claiming $50,000 for pain and suffering, $90,000 for loss of income, and $70,000 in punitive damages. Amber's insurance company will not pay punitive damages; therefore, Amber has offered to settle the case for $100,000 for pain and suffering, $90,000 for loss of income, and nothing for punitive damages. Khalid is in the 35% marginal tax bracket. What is the after-tax difference to Khalid between Khalid's original claim and Amber's offer? a. Amber's offer is $20,000 less. ($50,000 + $90,000 + $70,000 - $100,000 - $90,000). b. Amber's offer is $7,000 less. [($50,000 + $90,000 + $70,000 - $100,000 - $90,000) × .35)]. c. Amber's offer is $4,500 more. {$190,000 - ($50,000 + $90,000) + [$70,000 × (1 - .35)]}. d. Amber's offer is $22,000 more. [($190,000 - $210,000) + ($120,000 × .35)]. e. None of these.

ANSWER: c RATIONALE: Only the punitive damages are taxable. The after-tax proceeds from the amount of punitive damages claimed, (1 - .35) × $70,000 = $45,500.

Raul is married and files a joint tax return. His current investment interest expense of $95,000 is related to a loan used to purchase a parcel of unimproved land being held as an investment. Income from investments [dividends (not qualified) and interest] total $18,000. Raul paid and deducted $5,000 of real estate taxes on the unimproved land. He also has a $4,500 net long-term capital gain from the sale of another parcel of unimproved land. Raul's maximum investment interest deduction for the year is: a. $95,000. b. $18,000. c. $17,500. d. $13,000. e. None of the above.

ANSWER: c RATIONALE: Raul's net investment income is computed as follows: Income from investments: Interest and dividends $18,000 Long-term capital gain* 4,500 Less: Investment expenses (real estate taxes) (5,000) Net investment income $17,500 *Net capital gain generally is not included in investment income. In order to maximize his current investment interest deduction, Raul may elect to treat capital gain as investment income. If this election is made, however, the beneficial tax rate on capital gains will not apply. Raul's current investment interest expense deduction is limited to $17,500, the amount of net investment income, and $77,500 would be disallowed: Total investment interest expense $95,000 Less: Net investment interest income (17,500) Investment interest disallowed in the current year $77,500 The amount of investment interest disallowed carries over and becomes investment interest expense in the subsequent year (subject to the net investment income limitation).

Doctor and dentist bills for Richard 5,100 Prescribed medicines for Richard 830 Nonprescribed insulin for Richard 960 Derrick and Jane would qualify as Richard's dependents except that they file a joint return. Richard's medical insurance policy does not cover them. Richard filed a claim for $4,800 of his own expenses with his insurance company in November 2018 and received the reimbursement in January 2019. What is Richard's maximum allowable medical expense deduction for 2018? a. $0 b. $7,090 c. $10,340 d. $20,090 e. None of the above

ANSWER: c RATIONALE: Richard's medical expense deduction is $10,340, determined as follows: Medical insurance premiums $ 5,300 Doctor and dentist bills for Derrick and Jane 7,900 Doctor and dentist bills for Richard 5,100 Prescribed medicines for Richard 830 Nonprescribed insulin for Richard 960 Total medical expenses $20,090 Less: 7.5% of $130,000 (AGI) (9,750) Deductible portion of medical expenses $ 10,340 Although Derrick and Jane cannot be claimed as Richard's dependents, they could have been had they not filed a joint return. Therefore, they qualify for the medical expense deduction. Insulin is an exception to the rule that nonprescribed drugs do not qualify as medical expenses. The insurance recovery was not received until 2019. Therefore, it has no effect on the medical expense deduction for 2018.

Green, Inc., provides group term life insurance for all of its employees. The coverage equals twice the employee's annual salary. Sam, a vice-president, worked all year for Green, Inc., and received $200,000 of coverage for the year at a cost to Green of $1,500. The Uniform Premiums (based on Sam's age) are $.25 per month for $1,000 of protection. How much must Sam include in gross income this year? a. $0. b. $375. c. $450. d. $600. e. None of these.

ANSWER: c RATIONALE: Sam must include in gross income the uniform premium amount for his coverage in excess of $50,000, or $450 [(.25)(12) × ($200,000 - $50,000)/$1,000].

Gold Company was experiencing financial difficulties, but was not bankrupt or insolvent. The National Bank, which held a mortgage on other real estate owned by Gold, reduced the principal from $110,000 to $85,000. The bank had made the loan to Gold when it purchased the real estate from Silver, Inc. Pink, Inc., the holder of a mortgage on Gold's building, agreed to accept $40,000 in full payment of the $55,000 due. Pink had sold the building to Gold for $150,000 that was to be paid in installments over 8 years. As a result of the above, Gold must: a. Include $40,000 in gross income. b. Reduce the basis in its assets by $40,000. c. Include $25,000 in gross income and reduce its basis in its assets by $15,000. d. Include $15,000 in gross income and reduce its basis in the building by $25,000. e. None of these.

ANSWER: c RATIONALE: The $15,000 reduction in the mortgage held by Pink is an adjustment to the cost of the building, because the debt was owed to the party who sold the property to the taxpayer; in effect, the purchase price was adjusted. The $25,000 reduction by the bank is includible in gross income.

Under the terms of a divorce agreement entered into in 2018, Kim was to pay her husband Tom $7,000 per month in alimony. Kim's payments will be reduced to $3,000 per month when their 9 year-old son becomes 21. The husband has custody of their son. For a twelve-month period, Kim can deduct from gross income (and Tom must include in gross income): a. $60,000. b. $48,000. c. $36,000. d. $0. e. None of these

ANSWER: c RATIONALE: The $4,000 per month ($48,000) for child support is not deductible by Kim. The alimony of $3,000 per month ($36,000) is deductible by Kim

Under the terms of a divorce agreement entered into in 2017, Lanny was to pay his wife Joyce $2,000 per month in alimony and $500 per month in child support. For a twelve-month period, Lanny can deduct from gross income (and Joyce must include in gross income): a. $0. b. $6,000. c. $24,000. d. $30,000. e. None of these

ANSWER: c RATIONALE: The $500 per month for child support is not deductible by Lanny

The Hutters filed a joint return for 2018. They provide more than 50% of the support of Carla, Ellie, and Aaron. Carla (age 18) is a cousin and earns $2,800 from a part-time job. Ellie (age 25) is their daughter and is a full-time law student. She received a $7,500 scholarship for tuition from her law school. Aaron is a brother who is a citizen of Israel but resides in France. Carla and Ellie live with the Hutters. How many dependents can the Hutters claim? a. None b. One c. Two d. Three

ANSWER: c RATIONALE: The Hutters can claim two dependents. Carla is a qualifying relative and passes the gross income test. Ellie is not a qualifying child due to age (not under 24) but is a qualifying relative. Ellie also meets the gross income test since this type of scholarship is nontaxable. Aaron meets neither the residency nor citizenship requirement.

Jena is a full-time undergraduate student at State University and qualifies as a dependent of her parents. Her only source of income is a $10,000 athletic scholarship ($1,000 for books, $5,500 tuition, $500 student activity fee, and $3,000 room and board). Jena's gross income for the year is: a. $10,000. b. $4,000. c. $3,000. d. $500. e. None of these.

ANSWER: c RATIONALE: The portion included in gross income is $3,000 for room and board. The books ($1,000), tuition ($5,500), and student activity fee ($500) qualify for exclusion. The fact that the scholarship is received for athletic, rather than academic, achievement is irrelevant, because the payments are to further the student's education.

Maroon Corporation expects the employees' income tax rates to increase next year. The employees use the cash method. The company presently pays on the last day of each month. The company is considering changing its policy so that the December salaries will be paid on the first day of the following year. What would be the effect on an employee of the proposed change in company policy for paying its salaries beginning December 2018? a. The employee would be required to recognize the income in December 2018 because it is constructively received at the end of the month. b. The employee would be required to recognize the income in December 2018 because the employee has a claim of right to the income when it is earned. c. The employee will not be required to recognize the income until it is received, in 2019. d. The employee can elect to either include the pay in 2018 or 2019. e. None of these.

ANSWER: c RATIONALE: The cash basis employees do not recognize the income until it is actually or constructively received. If the employer will not pay until January 2019, the employee has not constructively received the income in 2018, nor has the employee received the income under a claim of right in that year.

Turner, a successful executive, is negotiating a compensation plan with his potential employer. The employer has offered to pay Turner a $600,000 annual salary, payable at the rate of $50,000 per month. Turner counteroffers to receive a monthly salary of $40,000 ($480,000 annually) and a $180,000 bonus in 5 years when Turner will be age 65. a. If the employer accepts Turner's counteroffer, Turner will recognize $660,000 at the time the offer is accepted. b. If the employer accepts Turner's counteroffer, Turner will recognize as gross income $55,000 per month [($480,000 + $180,000)/12]. c. If the employer accepts Turner's counteroffer, Turner will recognize $40,000 income each month for the year and $180,000 in year 5. d. If the employer accepts Turner's counteroffer, Turner must recognize imputed interest income on the $180,000 to be received in 5 years. e. None of these.

ANSWER: c RATIONALE: The constructive receipt doctrine does not apply to the negotiations. Therefore, Turner will include the salary and bonus in his gross income in the tax year received in accordance with the negotiated contract.

Ramon incurred $83,100 of interest expense related to his investments this year. His investment income included $34,500 of interest and a $37,500 net capital gain on the sale of securities. Ramon has asked you to compute the amount of his deduction for investment interest, taking into consideration any options he might have. What is the maximum amount of Ramon's investment interest expense deduction in the current year? a. $19,500. b. $34,500. c. $72,000. d. $83,100. e. None of the above.

ANSWER: c RATIONALE: The deduction for investment interest is limited to the amount of net investment income reported. If Ramon elects to treat the net capital gain as investment income for purposes of computing the investment interest expense limitation, the deduction will be $72,000 ($34,500 interest + $37,500 net capital gain). If Ramon elects to include the capital gains as investment income, the $37,500 net capital gain will not qualify for beneficial alternative tax rate treatment. Therefore, he must decide between the tax benefit of an additional deduction of $37,500 versus the benefit of the reduced tax rates applicable to net capital gain

George, an unmarried cash basis taxpayer, received the following amounts during 2018: Interest on savings accounts $2,000 Interest on a State tax refund 600 Interest on City of Salem school bonds 350 Interest portion of proceeds of a 5% bank certificate of deposit purchased on July 1, 2017, and matured on June 30, 2018 250 Dividends on USG common stock 300 What amount should George report as gross income from dividends and interest for 2018? a. $2,300. b. $2,550. c. $3,150. d. $3,500. e. None of these

ANSWER: c RATIONALE: The interest on the Salem school bonds of $350 is tax exempt, but the interest on the State tax refund of $600 is taxable. Since the life of the certificate of deposit was not more than 1 year, the OID rules do not apply and, therefore, all of the interest is taxable in 2018. Thus, the gross income is $3,150 ($2,000 + $600 + $250 + $300)

During the year, Kim sold the following assets: business auto for a $1,000 loss, stock investment for a $1,000 loss, and pleasure yacht for a $1,000 loss. Presuming adequate income, how much of these losses may Kim claim? a. $0. b. $1,000. c. $2,000. d. $3,000. e. None of these.

ANSWER: c RATIONALE: The loss on the business auto of $1,000 is an ordinary loss, while the loss on the stock investment of $1,000 is a capital loss. The loss on the yacht of $1,000 is personal and, therefore, cannot be deducted.

Betty purchased an annuity for $24,000 in 2018. Under the contract, Betty will receive $300 each month for the rest of her life. According to the actuarial estimates, Betty will live to receive 96 payments and will receive a 3% return on her original investment. a. If Betty collects $3,000 in 2018, her gross income is $630 (.03 × $21,000). b. Betty has no gross income until she has collected $24,000. c. If Betty lives to collect more than 96 payments, all of the amounts collected after the 96th payment must be included in taxable income. d. If Betty lives to collect only 60 payments before her death, she will report a $6,000 loss from the annuity [$24,000 - (60 × $300) = $6,000] on her final return. e. None of these.

ANSWER: c RATIONALE: The options other than C are simply contrary to the scheme provided in the Code for the taxation of annuities. If Betty dies after collecting only 60 payments (before she has recovered all of her investment), a loss can be claimed on her final return, but the loss is the difference between her original cost and the capital she recovered prior to her death. Therefore, d. is incorrect.

Ben was diagnosed with a terminal illness. His physician estimated that Ben would live no more than 18 months. After he received the doctor's diagnosis, Ben cashed in his life insurance policy and used the proceeds to take a trip to see relatives and friends before he died. Ben had paid $12,000 in premiums on the policy, and he collected $50,000, the cash surrender value of the policy. Henry enjoys excellent health, but he cashed in his life insurance policy to purchase a new home. He had paid premiums of $12,000 and collected $50,000 from the insurance company. a. Neither Ben nor Henry is required to recognize gross income. b. Both Ben and Henry must recognize $38,000 ($50,000 - $12,000) of gross income. c. Henry must recognize $38,000 ($50,000 - $12,000) of gross income, but Ben does not recognize any gross income. d. Ben must recognize $38,000 ($50,000 - $12,000) of gross income, but Henry does not recognize any gross income. e. None of these.

ANSWER: c RATIONALE: The redemption of the policy by Ben qualifies as an accelerated death benefit paid to a terminally ill policy holder. The exclusion applies regardless of how the insurance proceeds are used. Thus, the realized gain of $38,000 is excluded from Ben's gross income.

Thelma and Mitch were divorced in 2017. The couple had a joint brokerage account that included stocks with a basis of $600,000 and a fair market value of $1,000,000. Under the terms of the divorce agreement, Mitch would receive the stocks and Mitch would pay Thelma $100,000 each year for 6 years, or until Thelma's death, whichever should occur first. Thelma and Mitch lived apart when the payments were made by Mitch. Mitch paid the $600,000 to Thelma over the six-year period. The divorce agreement did not contain the word "alimony." Then, Mitch sold the stocks for $1,300,000. Mitch's recognized gain from the sale is: a. $0. b. $1,000,000 ($1,300,000 - $300,000). c. $700,000 ($1,300,000 - $600,000). d. $300,000 ($1,300,000 - $1,000,000). e. None of these.

ANSWER: c RATIONALE: The stocks were marital property. When the marital property was distributed to Mitch, his basis in the stocks became the same as the couple's basis of $600,000. Therefore, his gain on the sale was $700,000 ($1,300,000 - $600,000).

Doug and Pattie received the following interest income in the current year: Savings account at Greenbacks Bank $4,000 United States Treasury bonds 250 Interest on State of Iowa bonds 200 Interest on Federal tax refund 150 Interest on state income tax refund 75 Greenbacks Bank also gave Doug and Pattie a cellular phone (worth $100) for opening the savings account. What amount of interest income should they report on their joint income tax return? a. $4,775. b. $4,675. c. $4,575. d. $4,300. e. None of these.

ANSWER: c RATIONALE: The value of the cell phone is treated as a payment for the use of Doug and Pattie's money, and therefore is interest income. Only the interest on State of Iowa bonds may be excluded from their gross income.

On January 5, 2018, Tim purchased a bond paying interest at 6% for $30,000. On March 31, 2018, he gave the bond to Jane. The bond pays $1,800 interest on December 31. Tim and Jane are cash basis taxpayers. When Jane collects the interest in December 2018: a. Tim must include all of the interest in his gross income. b. Jane must report $1,800 gross income for 2018. c. Jane reports $1,350 of interest income in 2018, and Tim reports $450 of interest income in 2018. d. Jane reports $450 of interest income in 2018, and Tim reports $1,350 of interest income in 2018. e. None of these is correct.

ANSWER: c RATIONALE: Tim held the bond for 3 months before he gave it to Jane, who held the bond for the other 9 months that the interest accrued. Therefore, Tim must recognize $450 (3/12 × $1,800), and Jane must recognize $1,350 (9/12 × $1,800).

Tony, age 15, is claimed as a dependent by his grandmother. During 2018, Tony had interest income from Boeing Corporation bonds of $1,000 and earnings from a part-time job of $800. Tony's taxable income is: a. $1,800. b. $1,800 - $800 - $1,050 = ($50). c. $1,800 - $1,150 = $650. d. $1,800 - $1,050 = $750. e. None of these.

ANSWER: c RATIONALE: Tony's standard deduction of $1,150 ($800 + $350) partially offsets his gross income of $1,800, resulting in taxable income of $650.

Kyle and Liza are married and under 65 years of age. During 2018, they furnish more than half of the support of their 19-year old daughter, Kendra, who lives with them. She graduated from high school in May 2017. Kendra earns $15,000 from a part-time job, most of which she sets aside for future college expenses. Kyle and Liza also provide more than half of the support of Kyle's cousin who lives with them. Liza's father, who died on January 3, 2018, at age 90, has for many years qualified as their dependent. How many dependents can Kyle and Liza claim? a. None b. One c. Two d. Three

ANSWER: c RATIONALE: Two (Kyle's cousin, and Liza's father). Kendra cannot be claimed because she is not a qualifying child and is subject to the gross income test. Kyle's cousin does not meet the relationship test but is a member of their household. It is assumed that Liza's father, as was true in the past, qualified as a dependent up to the point of death.

Heather's interest and gains on investments for the current year are as follows: Interest on Madison County school bonds $600 Interest on U.S. government bonds 700 Interest on a Federal income tax refund 200 Gain on the sale of Madison County school bonds 500 Heather's adjusted gross income from the above is: or; Heather must report gross income in the amount of: a. $2,000. b. $1,800. c. $1,400. d. $1,300. e. None of these.

ANSWER: c RATIONALE: While the $600 interest on the Madison County school bonds is tax-exempt, the $500 gain on the sale of these bonds is included in Heather's gross income. The $200 interest on the Federal income tax refund and the $700 interest on U.S. government bonds are included in gross income. Thus, the includible amount is $1,400 ($700 + $200 + $500).

If a vacation home is determined to be a personal/rental use residence, which of the following statements is correct? a. All rental income is included in gross income. b. All rental related expenses that are deductible are classified as deductions from AGI. c. Expenses must be allocated between rental and personal use. d. Only a. and c. are correct. e. a., b., and c. are correct

ANSWER: d RATIONALE: The allowable deductions are classified as deductions for AGI.

Nancy paid the following taxes during the year: Tax on residence (for the period from March 1 through August 31) $5,250 State motor vehicle tax (based on the value of the personal use automobile) 430 State sales tax 3,500 State income tax 3,050 Nancy sold her personal residence on June 30 of this year under an agreement in which the real estate taxes were not prorated between the buyer and the seller. What amount qualifies as a deduction from AGI for Nancy? a. $9,180 b. $9,130 c. $7,382 d. $5,382 e. None of the above

ANSWER: c RATIONALE: [(121 days/184 days × $5,250) + $430 + $3,500] = $7,382. Nancy chooses to claim a deduction for the (higher) state sales tax rather than the state income tax

Frank established a Roth IRA at age 25 and contributed a total of $131,244 to it over 38 years. The account is now worth $376,000. How much of these funds can Frank withdraw tax-free? a. $0 b. $131,244 c. $244,756 d. $376,000 e. None of the above

ANSWER: d RATIONALE: $376,000. Assuming that Frank met the AGI limitations at the time of his contributions, all of the funds may be withdrawn tax-free. He satisfies the five-year holding period requirement for a Roth IRA and is over age 59 1/2 at the time of the distribution

During 2018, Sandeep had the following transactions: Salary $ 80,000 Interest income on City of Baltimore bonds 1,000 Damages for personal injury (car accident) 100,000 Punitive damages (same car accident) 200,000 Cash dividends from Chevron Corporation stock 7,000 Sandeep's AGI is: a. $185,000. b. $187,000. c. $285,000. d. $287,000. e. $387,000.

ANSWER: d RATIONALE: $80,000 (salary) + $200,000 (punitive damages) + $7,000 (cash dividends) = $287,000. The damages from personal injury and the municipal bond interest are nontaxable exclusions.

In 2018, Nai-Yu had the following transactions: Salary $90,000 Short-term capital gain from a stock investment 4,000 Moving expense to change jobs (11,000) Received repayment of $20,000 loan she made to her sister in 2014 (includes no interest) 20,000 State income taxes (5,000) Nai-Yu's AGI is: a. $114,000. b. $103,000. c. $98,000. d. $94,000. e. $83,000.

ANSWER: d RATIONALE: $90,000 (salary) + $4,000 (gain on stock investment) = $94,000. The loan repayment of $20,000 is a return of capital and has no effect on gross income. The moving expenses are not deductible. State income taxes paid are a deduction from AGI (or a standard deduction) and has no impact on the determination of AGI.

Which, if any, of the following is a correct statement relating to the kiddie tax in 2018? a. If the parents are divorced, the income of the noncustodial parent is used to determine the allocable parental tax. b. The components for the application of the kiddie tax are not subject to adjustment for inflation. c. If the kiddie tax applies, the parents must include the income of the child on their own income tax return. d. The kiddie tax does not apply if both parents of the child are deceased. e. None of these.

ANSWER: d RATIONALE: Adjustments are made to reflect inflation—for 2018 the reduction to arrive at net unearned income is $2,100 ($1,050 + $1,050) (choice b.). Choices a. and c. are not correct; beginning in 2018, the kiddie tax is assessed on the child (and the parent's taxable income is ignored).

Albert had a terminal illness which required almost constant nursing care for the remaining two years of his estimated life, according to his doctor. Albert had a life insurance policy with a face amount of $100,000. Albert had paid $25,000 of premiums on the policy. The insurance company has offered to pay him $80,000 to cancel the policy, although its cash surrender value was only $55,000. Albert accepted the $80,000. Albert used $15,000 to pay his medical expenses. Albert made a miraculous recovery and lived another 20 years. As a result of cashing in the policy: a. Albert must recognize $55,000 of gross income, but he has $15,000 of deductible medical expenses. b. Albert must recognize $65,000 ($80,000 - $15,000) of gross income. c. Albert must recognize $40,000 ($80,000 - $25,000 - $15,000) of gross income. d. Albert is not required to recognize any gross income because of his terminal illness. e. None of these.

ANSWER: d RATIONALE: Albert's realized gain is $55,000 ($80,000 - $25,000). However, Albert was terminally ill and, therefore, can qualify for the accelerated death benefit exclusion for his life insurance policy

Which of the following expenses, if any, qualify as deductible? a. Contributions to a Coverdell Education Savings Account (CESA). b. Contributions to a qualified tuition program (§ 529 plan). c. Job hunting expense of FBI agent who applies for the job of city manager of Beaumont (TX). d. Contribution to a traditional IRA. e. None of these.

ANSWER: d RATIONALE: Although the benefits paid out are nontaxable, neither the contribution to CESAs (choice a.) nor § 529 plans (choice b.) are deductible. Since being an FBI agent and a city manager do not appear to be in the same trade or business (law enforcement), the job hunting expense does not qualify (choice c.). Further, even if allowed, the job hunting expenses would be a miscellaneous itemized deduction (and not allowed from 2018 through 2025).

During 2018, Trevor has the following capital transactions: LTCG $ 6,000 Long-term collectible gain 2,000 STCG 4,000 STCL 10,000 After the netting process, the following results: a. Long-term collectible gain of $2,000. b. LTCG of $6,000, Long-term collectible gain of $2,000, and a STCL of $6,000. c. LTCG of $6,000, Long-term collectible gain of $2,000, and a STCL carryover to 2019 of $3,000. d. LTCG of $2,000. e. None of these.

ANSWER: d RATIONALE: First, the STCG and STCL are combined, resulting in a STCL of $6,000. Of this STCL, $2,000 is applied against the collectible gain of $2,000, and the $4,000 balance is applied against the LTCG of $6,000. The result is a LTCG of $2,000.

Kristen's employer owns its building and provides parking space for its employees. The value of the free parking is $150 per month. Karen's employer does not have parking facilities, but reimburses its employee for the cost of parking in a nearby garage, up to $150 per month. a. Kristen and Karen must recognize gross income from the parking services. b. Kristen can exclude the employer provided parking from gross income, but Karen must include her reimbursement in gross income. c. Kristen must include the value of the employer provided parking from her gross income, but Karen can exclude her reimbursement from gross income. d. Neither Kristen nor Karen is required to include the cost of parking in gross income. e. None of these

ANSWER: d RATIONALE: As a qualified transportation fringe, both Kristen and Karen's benefit can be excluded from gross income. Karen's reimbursement is less than the maximum exclusion amount allowed. Likewise, the value of Karen's free parking is less than the maximum exclusion amount allowed.

Mary establishes a Roth IRA at age 50 and contributes the maximum amount per year to the Roth IRA for 15 years. The account is now worth $199,000, consisting of $75,000 in contributions plus $124,000 in accumulated earnings. How much can Mary withdraw tax-free? a. $0 b. $75,000 c. $124,000 d. $199,000 e. None of the above

ANSWER: d RATIONALE: Assuming that Mary meets the income limitation at the time of the contributions to the Roth IRA, all of the funds may be withdrawn tax-free. She satisfies the fiveyear holding period for a Roth IRA and is over age 59 1/2 at the time of the distribution.

Priscella pursued a hobby of making bedspreads in her spare time. Her AGI before considering the hobby is $40,000. During 2018 she sold the bedspreads for $10,000. She incurred expenses as follows: Supplies $4,000 Interest on loan to get business started 500 Advertising 6,500 Assuming that the activity is deemed a hobby, how should she report these items on her tax return? a. Include $10,000 in income and deduct $11,000 for AGI. b. Ignore both income and expenses since hobby losses are disallowed. c. Include $10,000 in income, deduct nothing for AGI, and claim $11,000 of the expenses as itemized deductions. d. Include $10,000 in income and deduct nothing. e. None of the above.

ANSWER: d RATIONALE: Beginning in 2018 expenses related to a hobby are not deductible.

Flora Company owed $95,000, a debt incurred to purchase land that serves as security for the debt. a. If Flora had borrowed the funds from a bank, the bank accepts $85,000 in full payment of the debt, and Flora is solvent after the transfer, Flora does not recognize income, but the company must reduce the cost of the land by $10,000. b. If Flora had borrowed the funds from a bank, and the bank accepts $85,000 in full payment of the debt, when the value of the property is $80,000, Flora can deduct a loss. c. If Flora transfers to the bank other property, with a basis of $90,000 and a fair market value of $95,000, in full payment of the debt, Flora can recognize a $5,000 loss. d. If the $95,000 is owed to the person who sold the property to Flora, and the creditor accepts $85,000 in full payment for the debt, Flora does not recognize gain but must reduce its basis in the land. e. None of these.

ANSWER: d RATIONALE: Choice a. is incorrect because Flora reduced a $95,000 liability with a payment of $85,000, and therefore its net worth increased. Choice b. is incorrect because Flora did not sell the property. Choice c. is incorrect because the transfer is treated as a sale of the property for $95,000 when its basis is $90,000. Choice d. is correct because the creditor was the original seller of the property.

A U.S. citizen worked in a foreign country for the period July 1, 2017 through August 1, 2018. Her salary was $10,000 per month. Also, in 2017 she received $5,000 in dividends from foreign corporations (not qualified dividends). No dividends were received in 2018. Which of the following is correct? a. The taxpayer cannot exclude any of the income because she was not present in the foreign country more than 330 days in either 2017 or 2018. b. The taxpayer can exclude a portion of the salary from U.S. gross income in 2017 and 2018, and all of the dividend income. c. The taxpayer can exclude from U.S. gross income $60,000 salary in 2017, but in 2018 the taxpayer will exceed the twelve month limitation and, therefore, all of the 2018 compensation must be included in gross income. All of the dividends must be included in 2017 gross income. d. The taxpayer must include the dividend income of $5,000 in 2017 gross income, but the taxpayer can exclude a portion of the compensation income from U.S. gross income in 2017 and 2018. e. None of these

ANSWER: d RATIONALE: Choice a. is incorrect because the taxpayer was in the foreign country more than 330 days in a 12 month period. The annual statutory limit on the foreign earned income exclusion is $102,100 in 2017 and $103,900 in 2018, and the exclusion is prorated on a daily basis; therefore, choice c., is incorrect. Choice b. is incorrect with regard to the dividends received in that dividends received cannot be excluded from gross income.

Natalie is married to Chad, who abandoned her in early June of 2018. She has not seen or communicated with him since then. She maintains a household in which she and her two dependent children live. Which of the following statements about Natalie's filing status in 2018 is correct? a. Natalie can use the rates for single taxpayers. b. Natalie can file a joint return with Chad. c. Natalie can file as a surviving spouse. d. Natalie can file as a head of household. e. None of these statements is appropriate.

ANSWER: d RATIONALE: Natalie meets the "abandoned spouse" rules. Therefore, she can file as a head of household

The employees of Mauve Accounting Services are permitted to use the copy machine for personal purposes, provided the privilege is not abused. Ed is the president of a civic organization and uses the copier to make several copies of the organization's agenda for its meetings. The copies made during the year would have cost $150 at a local office supply. a. Ed must include $150 in his gross income. b. Ed may exclude the cost of the copies as a no-additional cost fringe benefit. c. Ed may exclude the cost of the copies only if the organization is a client of Mauve. d. Ed may exclude the cost of the copies as a de minimis fringe benefit. e. None of these.

ANSWER: d RATIONALE: Choice b. is incorrect because the employer does incur additional costs. Choice c. is incorrect because there are other circumstances where the cost of the copies would not result in income to Ed. "Occasional copies" is a frequently permitted de minimis fringe benefit.

. Jordan performs services for Ryan. Which, if any, of the following factors indicate that Jordan is an independent contractor, rather than an employee? a. Ryan sets the work schedule. b. Ryan provides the tools used. c. Jordan follows a specific set of instructions from Ryan to complete tasks. d. Jordan is paid based on tasks performed. e. None of these

ANSWER: d RATIONALE: Choices a., b., and c. reflect employee status.

Emily, who lives in Indiana, volunteered to travel to Louisiana in March to work on a home-building project for Habitat for Humanity (a qualified charitable organization). She was in Louisiana for three weeks. She normally makes $500 per week as a carpenter's assistant and plans to deduct $1,500 as a charitable contribution. In addition, she incurred the following costs in connection with the trip: $600 for transportation, $1,200 for lodging, and $400 for meals. What is Emily's deduction associated with this charitable activity? a. $600 b. $1,200 c. $1,800 d. $2,200 e. $3,700

ANSWER: d RATIONALE: Emily cannot deduct the estimated value of $1,500 of her contributed services. However, she can deduct out-of-pocket costs of $2,200 ($600 for transportation, $1,200 for lodging, and $400 for meals).

In which, if any, of the following situations may the individual not be claimed as a dependent of the taxpayer? a. A former spouse who lives with the taxpayer (divorce took place last year). b. A stepmother who does not live with the taxpayer. c. A married daughter who lives with the taxpayer. d. A half-brother who does not live with the taxpayer and is a citizen and resident of Honduras. e. A cousin who lives with the taxpayer.

ANSWER: d RATIONALE: Except in the year of divorce, a former spouse can qualify under the member of the household test (choice a.). The stepmother meets the relationship test (choice b.). A married daughter can be claimed as long as she does not violate the joint return test (choice c.). In the case of the half brother, only being a citizen or resident of the U.S. can satisfy the residency test (choice d.). A cousin does not satisfy the relationship test so must be a member of the household (choice e.).

Which of the following legal expenses are deductible for AGI in 2018? a. Incurred in connection with a trade or business. b. Incurred in connection with rental or royalty property held for the production of income. c. Incurred for tax advice relative to the preparation of an individual's income tax return. d. Only a. and b. qualify. e. a., b., and c. qualify.

ANSWER: d RATIONALE: Expenses paid in 2018 for tax advice relative to the preparation of an individual's income tax return are not deductible.

Brad, who would otherwise qualify as Faye's dependent, had gross income of $9,000 during the year. Faye, who had AGI of $120,000, paid the following medical expenses this year: Cataract operation for Brad $ 5,400 Brad's prescribed contact lenses 1,800 Faye's doctor and dentist bills 12,600 Prescribed drugs for Faye 2,550 Total $22,350 Faye has a medical expense deduction of: a. $3,150 b. $4,950 c. $10,350 d. $13,350 e. None of the above

ANSWER: d RATIONALE: Faye may claim the medical expenses she paid on behalf of Brad (cataract operation and contact lenses), even though Brad cannot be claimed as a dependent. This exception applies if the gross income and/or joint return tests are the only reasons a person cannot be otherwise claimed as a dependent. Her deduction is $13,350 [$22,350 - ($120,000 × 7.5%)].

The First Chance Casino has gambling facilities, a bar, a restaurant, and a hotel. All employees are allowed to obtain food from the restaurant at no charge during working hours. In the case of the employees who operate the gambling facilities, bar, and restaurant, 60% of all of Casino's employees, the meals are provided for the convenience of the Casino. However, the hotel workers demanded equal treatment and therefore were also allowed to eat in the restaurant at no charge while they are at work. Which of the following is correct? a. All the employees are required to include the value of the meals in their gross income. b. Only the restaurant employees may exclude the value of their meals from gross income. c. Only the employees who work in gambling, the bar, and the restaurant may exclude the meals from gross income. d. All of the employees may exclude the value of the meals from gross income. e. None of these.

ANSWER: d RATIONALE: For more than 50% of the employees, the meals are furnished for the convenience of the employer; therefore, all of the meals are excludible.

The plant union is negotiating with the Eagle Company, which is on the verge of bankruptcy. Eagle has offered to pay for the employees' hospitalization insurance in exchange for a wage reduction. The employees each currently pay premiums of $4,000 a year for their insurance. Which of the following is correct: a. If an employee's wages are reduced by $5,000 and the employee is in the 24% marginal tax bracket, the employee would benefit from the offer. b. If an employee's wages are reduced by $4,000 and the employee is in the 12% marginal tax bracket, the employee would benefit from the offer. c. If an employee's wages are reduced by $6,000 and the employee is in the 35% marginal tax bracket, the employee would benefit from the offer. d. a., b., and c. e. None of these.

ANSWER: d RATIONALE: In all three cases, the reduction in after-tax pay of the employee will be less than the $4,000 value of the nontaxable insurance premiums to be paid by the employer. Reduction in Pay Marginal Tax Rate Reduction in Tax Reduction in After-tax Income a. $5,000 0.24 $1,200 $3,800 b. $4,000 0.12 $ 480 $3,520 c. $6,000 0.35 $2,100 $3,900

Jay, a single taxpayer, retired from his job as a public school teacher in 2018. He is to receive a retirement annuity of $1,200 each month and his life expectancy is 180 months. He contributed $36,000 to the pension plan during his 35- year career; so his adjusted basis is $36,000. Jay collected 192 payments before he died. What is the correct method for reporting the pension income? a. Since Jay is no longer working, none of the pension payments must be included in his gross income. b. The first $36,000 received is a nontaxable recovery of capital, and all subsequent annuity payments are taxable. c. The first $180,000 he receives is taxable and the last $36,000 is a nontaxable recovery of capital. d. All of the last 12 payments he received ($14,400) are taxable. e. None of these

ANSWER: d RATIONALE: Jay is collecting under an annuity contract and the cost must be allocated among the payments received on the basis of the cost/expected return, until the total cost has been recovered. Thus, for each annuity payment received for the 180-month period, $200 [$36,000/($1,200 × 180) × $1,200] is excluded from gross income and $1,000 is included in gross income. Any payments received after the 180-month period are included in his gross income.

In January, Lance sold stock with a cost basis of $26,000 to his brother, James, for $24,000, the fair market value of the stock on the date of sale. Five months later, James sold the same stock through his broker for $27,000. What is the tax effect of these transactions? a. Disallowed loss to James of $2,000; gain to Lance of $1,000. b. Disallowed loss to Lance of $2,000; gain to James of $3,000. c. Deductible loss to Lance of $2,000; gain to James of $3,000. d. Disallowed loss to Lance of $2,000; gain to James of $1,000. e. None of the above.

ANSWER: d RATIONALE: Lance's realized loss of $2,000 ($24,000 - $26,000) is disallowed. James may reduce his realized gain of $3,000 ($27,000 - $24,000) by Lance's disallowed loss of $2,000. So James' recognized gain is $1,000.

Which of the following miscellaneous expenses is deductible in 2018? a. Unreimbursed employee business expenses. b. Job hunting expenses. c. Union dues. d. Losses from Ponzi-type investment schemes. e. All of the above expenses are miscellaneous itemized deductions.

ANSWER: d RATIONALE: Losses from Ponzi-type investment schemes are miscellaneous itemized deductions, but are not subject to the 2%-of-AGI limitation. Items a., b., and c. are all subject to the 2%-of-AGI limitation and not deductible from 2018 through 2025.

Merrill is a participant in a SIMPLE § 401(k) plan, and he elects to contribute 4% of his $40,000 compensation to the account, while his employer contributes 3%. What amount will vest immediately, if any? a. $0 b. $1,200 c. $1,600 d. $2,800 e. None of above

ANSWER: d RATIONALE: Merrill has elected to contribute $1,600 ($40,000 × 4%) to his SIMPLE § 401(k) plan. His employer will contribute $1,200 ($40,000 × 3%). Both amounts will vest immediately.

The § 222 deduction for tuition and related expenses is available: a. Regardless of the amount of a taxpayer's MAGI. b. To cover room and board expenses to attend college. c. To a married taxpayer filing a separate return. d. Even if a taxpayer does claim the standard deduction. e. None of these.

ANSWER: d RATIONALE: No deduction at all is allowed if the taxpayer has AGI in excess of $80,000 ($160,000 in the case of a joint return) (choice a.). Section 222 does not cover room and board (choice b.). In order to claim a deduction under § 222, a married taxpayer must file a joint return (choice c.). Since § 222 provides for a deduction for AGI, it does not matter if a taxpayer claims the standard deduction.

Jack received a court award in a civil libel and slander suit against National Gossip. He received $120,000 for damages to his professional reputation, $100,000 for damages to his personal reputation, and $50,000 in punitive damages. Jack must include in his gross income as a damage award: a. $0. b. $100,000. c. $120,000. d. $270,000. e. None of these.

ANSWER: d RATIONALE: None of the damages received were the result of a physical personal injury or sickness and therefore the total amount received must be included in gross income. Even if the damages were the result of physical personal injury, the punitive damages would be included in his gross income.

Cory incurred and paid the following expenses in 2018: Tax return preparation fee $ 600 Moving expenses 2,000 Investment expenses 500 Expenses associated with rental property 1,500 Interest expense associated with loan to finance tax-exempt bonds 400 Calculate the amount that Cory can deduct (before any percentage limitations). a. $5,000. b. $4,600. c. $3,000. d. $1,500. e. None of the above.

ANSWER: d RATIONALE: Only the rental property expenses are deductible.

Which is not an advantage of a § 401(k) plan over a traditional IRA? a. Deduction limitation. b. Early withdrawal option without penalty. c. Loans from the plan. d. Record keeping. e. All of the above are advantages of § 401(k) plan

ANSWER: d RATIONALE: Record keeping is more detailed for § 401(k) plan

Sammy, age 31, is unmarried and is not an active participant in a qualified retirement plan. His modified AGI is $55,000 in 2018. The maximum amount that Sammy can deduct for a contribution to a traditional IRA is: a. $2,800. b. $3,500. c. $5,000. d. $5,500. e. None of the above.

ANSWER: d RATIONALE: Sammy can deduct $5,500 because he is not an active participant in a qualified retirement plan.

Sammy, a calendar year cash basis taxpayer who is age 66, has the following transactions in 2018: Salary from job $90,000 Alimony received from ex-wife 10,000 Medical expenses 7,000 Based on this information, Sammy has: a. AGI of $90,000. b. AGI of $95,000. c. AGI of $99,500. d. Deduction for medical expenses of $0. e. None of the above.

ANSWER: d RATIONALE: Sammy's AGI is calculated as follows: Salary from job $ 90,000 Alimony received from ex-wife 10,000 AGI $100,000 Sammy's deduction for medical expenses, an itemized deduction, is $0 [$7,000 - 7.5%($100,000)].

Harpreet, whose husband died in December 2017, maintains a household in which her dependent mother lives. Which (if any) of the following is her filing status for the tax year 2018? (Note: Harpreet is the executor of her husband's estate.) a. Single b. Married, filing separately c. Surviving spouse d. Head of household e. Married, filing jointly

ANSWER: d RATIONALE: She does not qualify for surviving spouse status in 2018.

Robyn rents her beach house for 60 days and uses it for personal use for 30 days during the year. The rental income is $6,000 and the expenses are as follows: Mortgage interest $9,000 Real estate taxes 3,000 Utilities 2,000 Maintenance 1,000 Insurance 500 Depreciation (rental part) 4,000 Using the IRS approach, total expenses that Robyn can deduct on her tax return associated with the beach house are: a. $0. b. $6,000. c. $8,000. d. $12,000. e. None of the above.

ANSWER: d RATIONALE: Since the property is classified as personal/rental use, the general rule is that the deductible expenses cannot exceed the gross income. Thus, under the general rule, the deductible expenses would be limited to $6,000. However, this ceiling does not apply to expenses that otherwise would be deductible as itemized deductions. Consequently, all of the mortgage interest and real estate taxes can be deducted ($9,000 + $3,000 = $12,000).

Swan Finance Company, an accrual method taxpayer, requires all of its customers to carry credit life insurance. If a customer dies, the company receives from the insurance company the balance due on the customer's loan. Ali, a customer, died owing Swan $1,500. The balance due included $200 accrued interest that Swan has included in income. When Swan collects $1,500 from the insurance company, Swan: a. Must recognize $1,500 income from the life insurance proceeds. b. Must recognize $1,300 income from the life insurance proceeds. c. Does not recognize income because life insurance proceeds are tax-exempt. d. Does not recognize income from the life insurance because the entire amount is a recovery of capital. e. None of these.

ANSWER: d RATIONALE: Swan has a basis in the receivable of $1,500, since it is an accrual method taxpayer.

The taxpayer's marginal federal and state tax rate is 25%. Which would the taxpayer prefer? a. $1.00 taxable income rather than $1.25 tax-exempt income. b. $1.00 taxable income rather than $.75 tax-exempt income. c. $1.25 taxable income rather than $1.00 tax-exempt income. d. $1.40 taxable income rather than $1.00 tax-exempt income. e. None of these.

ANSWER: d RATIONALE: The $1.40 of taxable income is greater after-tax than $1.00 in taxable income [(1 - .25)($1.40) = $1.05]. Choice b. is incorrect because the $1.00 of taxable income and $.75 of tax-exempt income are equal on an after-tax basis (1 - .25) × $1.00 = $.75. Choice c. is incorrect because (1 - .25) × $1.25 = $.9375.

Sharon made a $60,000 interest-free loan to her son, Todd, who used the money to start a new business. Todd's only sources of income were $25,000 from the business and $490 of interest on his checking account. The relevant Federal interest rate was 5%. Based on the above information: a. Todd's business net profit will be reduced by $3,000 (.05 × $60,000) of interest expense. b. Sharon must recognize $3,000 (.05 × $60,000) of imputed interest income on the below- market loan. c. Todd's gross income must be increased by the $3,000 (.05 × $60,000) imputed interest income on the below market loan. d. Sharon does not recognize any imputed interest income and Todd does not recognize any imputed interest expense. e. None of these is correct.

ANSWER: d RATIONALE: The $100,000 exception would apply. Sharon is not required to recognize imputed interest income because Todd's investment income is less than $1,000. Todd does not recognize any imputed interest expense.

Tim and Janet were divorced in 2018. Their only marital property was a personal residence with a value of $120,000 and cost of $50,000. Under the terms of the divorce agreement, Janet would receive the house and Janet would pay Tim $15,000 each year for 5 years, or until Tim's death, whichever should occur first. Tim and Janet lived apart when the payments were made to Tim. The divorce agreement did not contain the word "alimony." a. Tim must recognize a $35,000 [$60,000 - 1/2($50,000)] gain on the sale of his interest in the house. b. Tim does not recognize any income from the above transactions. c. Janet is not allowed any alimony deductions. d. Janet is allowed to deduct $15,000 each year for alimony paid. e. None of these.

ANSWER: d RATIONALE: The $15,000 cash payments meet all of the requirements for alimony treatment. Although the circumstances suggest that Janet is paying Tim for his share of the marital property, the agreement must specify that the payments are not alimony to avoid alimony treatment

Freddy purchased a certificate of deposit for $20,000 on July 1, 2018. The certificate's maturity value in two years (June 30, 2020) is $21,218, yielding 3% before-tax interest. a. Freddy must recognize $1,218 gross income in 2018. b. Freddy must recognize $1,218 gross income in 2020. c. Freddy must recognize $600 (.03 × $20,000) gross income in 2020. d. Freddy must recognize $300 (.03 × $20,000 × .5) gross income in 2018. e. None of these.

ANSWER: d RATIONALE: The 3% interest rate is applied to the $20,000 original investment in the first year $600 ($20,000 × 3%). The certificate was held for only 6 months in 2018; therefore, the interest income for 2018 is $300 ($600 × 6/12). Answers a., b., and c. are incorrect because these answers assume a method of allocating the income that differs from the effective interest method.

Under the original issue discount (OID) rules as applied to a three-year certificate of deposit: a. All of the income must be recognized in the year of maturity by a cash basis taxpayer. b. The OID will be included in gross income for the year of purchase. c. The interest income will be the same each year. d. The interest income will be greater in the third year than in the first year. e. None of these is correct.

ANSWER: d RATIONALE: The OID is amortized using the effective interest rate method. Because the principal amount is increased each year by the amount of the OID which is amortized, the total interest income increases each year. Thus, answers a., b., c., and e. are incorrect.

The Maroon & Orange Gym, Inc., uses the accrual method of accounting. The corporation sells memberships that entitle the member to use the facilities at any time. A one-year membership costs $480 ($480/12 = $40 per month); a two-year membership costs $720 ($720/24 = $30 per month). Cash payment is required at the beginning of the membership period. On July 1, 2018, the company sold a one-year membership and a two-year membership. For financial reporting purposes, Maroon reports the membership income ratably over the number of months involved. The company should report as gross income from the two contracts: a. $1,200 in 2018. b. $960 in 2018. c. $180 in 2020. d. $780 in 2019. e. None of these.

ANSWER: d RATIONALE: The accrual basis taxpayer can prorate the income from services in the first year, but must include the balance of the income on the contract in the following year. For the one-year membership, $240 was reported in 2018 ($40 x 6). For the two-year membership, $180 was reported in 2018 ($30 x 6). The balance of the contracts is reported in 2019 ($480 + $720 - $240 - $180 = $780).

Office Palace, Inc., leased an all-in-one printer to a new customer, Ashley, on December 27, 2018. The printer was to rent for $600 per month for a period of 36 months beginning January 1, 2019. Ashley was required to pay the first and last month's rent at the time the lease was signed. Ashley was also required to pay a $1,500 damage deposit. Office Palace must recognize as income for the lease: a. $0 in 2018, if Office Palace is an accrual basis taxpayer. b. $7,800 in 2019, if Office Palace is a cash basis taxpayer. c. $2,700 in 2018, if Office Palace is a cash or accrual basis taxpayer. d. $1,200 in 2018, if Office Palace is a cash or accrual basis taxpayer. e. None of these.

ANSWER: d RATIONALE: The company is required to recognize the $1,200 (January 2018 and December 2021 rent) in 2018 because prepaid income from rents is ineligible for deferral. The damage deposit of $1,500 is not income.

The effects of a below-market loan for $100,000 made by a corporation to its chief executive officer as an enticement to get him to remain with the company are: a. The corporation has imputed interest income and the employee is deemed to have received a gift. b. The corporation has imputed interest income and dividends paid. c. The employee has no income unless the funds are invested and produce investment income for the year. d. The employee has imputed compensation income and the corporation has imputed interest income. e. None of these.

ANSWER: d RATIONALE: The corporation made a compensation related loan. It has imputed interest income and compensation expense. The employee has imputed interest expense and compensation income. Since the loan is for $100,000, the $10,000 exception does not apply.

Chen incurred $58,500 of interest expense this year related to his investments. His investment income includes $15,000 of interest, $9,000 of qualified dividends, and a $22,500 net capital gain on the sale of securities. The maximum amount of Chen's investment interest expense deduction for the year is: a. $15,000. b. $24,000. c. $37,500. d. $46,500. e. None of the above

ANSWER: d RATIONALE: The deduction for investment interest is limited to the amount of net investment income reported. If Chen elects to treat the capital gain and qualified dividends as investment income, the deduction is $46,500 ($15,000 interest + $9,000 qualified dividends + $22,500 net capital gain). However, if he so elects, his capital gain and qualified dividends will not be eligible for beneficial capital gains tax rate treatment.

Employees of the Valley Country Club are allowed to use the golf course without charge before and after working hours on Mondays, when the number of players on the course is at its lowest. Tom, an employee of the country club played 40 rounds of golf during the year at no charge when the non-employee charge was $20 per round. a. Tom must include $800 in gross income. b. Tom is not required to include anything in gross income because it is a de minimis fringe benefit. c. Tom is not required to include the $800 in gross income because the use of the course was a gift. d. Tom is not required to include anything in gross income because this is a "no-additional-cost service" fringe benefit. e. None of these.

ANSWER: d RATIONALE: The golfing privileges are a "no-additional-cost service." The privilege is not a de minimis fringe because records of Tom's use is readily available at little or no cost

The annual increase in the cash surrender value of a life insurance policy: a. Is taxed when the individual dies and the heirs collect the insurance proceeds. b. Must be included in gross income each year under the original issue discount rules. c. Reduces the deduction for life insurance expense. d. Is not included in gross income each year because of the substantial restrictions on gaining access to the policy's value. e. None of these

ANSWER: d RATIONALE: The income has not been actually received and, because of the restrictions, is not constructively received.

Tonya is a cash basis taxpayer. In 2018, she paid state income taxes of $8,000. In early 2019, she filed her 2018 state income tax return and received a $900 refund. a. If Tonya itemized her deductions in 2018 on her Federal income tax return, she should amend her 2018 return and reduce her itemized deductions by $900. b. If Tonya itemized her deductions in 2018 on her Federal income tax return and her itemized deductions exceeded the standard deduction by at least $900, the refund will not affect her 2019 tax return. c. If Tonya itemized her deductions in 2018 on her Federal income tax return, she must amend her 2018 Federal income tax return and use the standard deduction. d. If Tonya itemized her deductions in 2018 on her Federal income tax return and her itemized deductions exceeded the standard deduction by more than $900, she must recognize $900 income in 2019 under the tax benefit rule. e. None of these

ANSWER: d RATIONALE: The income recognized from the state income tax refund results from the application of the tax benefit rule. In choice d., Tonya received a tax benefit from the payment; therefore, the refund was taxable in 2019.

A participant, who is age 38, in a cash or deferred arrangement plan [§ 401(k)] may contribute up to what amount in 2018? a. $12,000 b. $17,000 c. $17,500 d. $18,500 e. None of the above

ANSWER: d RATIONALE: The indexed amount for 2018 is $18,500. A taxpayer who is age 38 is not eligible for the $6,000 catch-up amount in 2018

Statutory employees: a. Report their expenses as miscellaneous itemized deductions. b. Include common law employees. c. Are subject to income tax withholdings. d. Claim their expenses as deductions for AGI. e. None of these.

ANSWER: d RATIONALE: Their expenses are for AGI deductions and reported on Schedule C, not as miscellaneous itemized deductions (choice a.). Although subject to Social Security tax, they are not subject to income tax withholdings (choice c.). Statutory employees are not common law employees (choice b.). Expenses are deductions for AGI (choice d.)

Iris collected $150,000 on her deceased husband's life insurance policy. The policy was purchased by the husband's employer under a group policy. Iris's husband had included $5,000 in gross income from the group term life insurance premiums during the years he worked for the employer. She elected to collect the policy in 10 equal annual payments of $18,000 each. a. None of the payments must be included in Iris's gross income. b. The amount she receives in the first year is a nontaxable return of capital. c. For each $18,000 payment that Iris receives, she can exclude $500 ($5,000/$180,000 × $18,000) from gross income. d. For each $18,000 payment that Iris receives, she can exclude $15,000 ($150,000/$180,000 × $18,000) from gross income. e. None of these.

ANSWER: d RATIONALE: The life insurance proceeds of $150,000 are excluded from Iris's gross income. The income portion of each annuity payment is $3,000 ($18,000 - $15,000 recovery of capital). The recovery of capital of each annuity payment is $15,000 [($150,000/$180,000) × $18,000].

Sharon had some insider information about a corporate takeover. She unintentionally informed a friend, who immediately bought the stock in the target corporation. The takeover occurred and the friend made a substantial profit from buying and selling the stock. The friend told Sharon about his stock dealings, and gave her a pearl necklace because she "made it all possible." The necklace was worth $10,000, but she already owned more jewelry than she desired. a. The necklace is a nontaxable gift received by Sharon because the friend was not legally required to make the gift. b. The value of the necklace is not included in Sharon's gross income unless she sells it. c. The value of the necklace is not included in Sharon's gross income because passing the information was an illegal act and the SEC can confiscate the necklace. d. The value of the necklace must be included in Sharon's gross income for the tax year it was received by her. e. None of these.

ANSWER: d RATIONALE: The necklace Sharon received from her friend was compensation for the information, rather than an amount received out of detached generosity. The fact the friend was not legally obligated to make the payment does not affect the outcome in this case.

Because Scott is three months delinquent on the mortgage payments for his personal residence, Jeanette (his sister) is going to cover the arrearage. Based on past experience, she does not expect to be repaid by Scott. Which of the following statements is correct? a. If Scott receives the money from Jeanette and pays the mortgage company, Jeanette can deduct the interest part. b. If Jeanette pays the mortgage company directly, neither Scott nor Jeanette can deduct the interest part. c. If Jeanette pays the mortgage company directly, she cannot deduct the interest part. d. Only b. and c. are correct. e. a., b., and c. are correct.

ANSWER: d RATIONALE: The obligation is that of Scott and not of Jeanette. If Scott pays his mortgage company, he can deduct the mortgage interest part of the payment. Under no circumstances can Jeanette deduct Scott's mortgage interest.

Turner, Inc., provides group term life insurance to the officers of the corporation only. Janet, a vice-president, received $450,000 of coverage for the year at a cost to Turner, Inc. of $5,600. The Uniform Premiums (based on Janet's age) are $15 a year for $1,000 protection. How much of this must Janet include in gross income this year? a. $0. b. $2,700. c. $5,600. d. $6,000. e. None of these.

ANSWER: d RATIONALE: The plan is discriminatory. Therefore, the employee's income is the greater of the amount from the IRS table (($15) × (450-50) = $6,000) or the employer's cost ($5,600).

The amount of Social Security benefits received by an individual that he or she must include in gross income: a. Is computed in the same manner as an annuity [exclusion = (cost/expected return) × amount received]. b. May not exceed the portion contributed by the employer. c. May not exceed 50% of the Social Security benefits received. d. May be zero or as much as 85% of the Social Security benefits received, depending upon the taxpayer's Social Security benefits and other income. e. None of these.

ANSWER: d RATIONALE: The taxable Social Security benefits are based on a formula intended to assure that low income taxpayers are not taxed on the Social Security benefits, but higher income taxpayers are taxed under the theory that, on the average the taxpayer will collect more from the Social Security program than he or she paid into the program.

As an executive of Cherry, Inc., Ollie receives a fringe benefit in the form of annual tuition scholarships of $10,000 to each of his three children. The scholarships are paid by the company on behalf of the children of key employees directly to each child's educational institution and are payable only if the student maintains a B average. a. The tuition payments of $30,000 may be excluded from Ollie's gross income as a scholarship. b. The tuition payments of $10,000 each must be included in the child's gross income. c. The tuition payments of $30,000 may be excluded from Ollie's gross income because the payments are for the academic achievements of the children. d. The tuition payments of $30,000 must be included in Ollie's gross income. e. None of these.

ANSWER: d RATIONALE: The tuition payments of $30,000 are compensation to Ollie since the awards are only to children of key employees.

Evan and Eileen Carter are husband and wife and file a joint return for 2018. Both are under 65 years of age. They provide more than half of the support of their daughter, Pamela (age 25), who is a full-time medical student. Pamela receives a $5,000 scholarship covering her tuition at college. They furnish all of the support of Belinda (Evan's grandmother), who is age 80 and lives in a nursing home. They also support Peggy (age 66), who is a friend of the family and lives with them. How many dependents may the Carters claim? a. None b. One c. Two d. Three

ANSWER: d RATIONALE: Three (Pamela, Belinda, and Peggy). Pamela is not a qualifying child—although a full-time student, she is not under age 24. Pamela does meet the qualifying relative category as the type of scholarship aid she receives is nontaxable (the gross income test is satisfied). Belinda is not a member of the household but satisfies the relationship test. Peggy does not satisfy the relationship test but is a member of the household.

Ellen, age 12, lives in the same household with her father, grandfather, and uncle. The cost of maintaining the household is provided by her grandfather (40%) and her uncle (60%). Disregarding tie-breaker rules, Ellen is a qualifying child as to: a. Only her father. b. Only her grandfather and uncle. c. Only her uncle. d. All parties involved (i.e., father, grandfather, and uncle). e. None of these

ANSWER: d RATIONALE: Under the abode and relationship tests, Ellen is a qualifying child as to all parties. The amount of support provided by each person is not relevant.

Paula is the sole shareholder of Violet, Inc. For 2018, she receives from Violet a salary of $300,000 and dividends of $100,000. Violet's taxable income for 2018 is $500,000. On audit, the IRS treats $100,000 of Paula's salary as unreasonable. Which of the following statements is correct? a. Paula's gross income will increase by $100,000 as a result of the IRS adjustment. b. Violet's taxable income will not be affected by the IRS adjustment. c. Paula's gross income will decrease by $100,000 as a result of the IRS adjustment. d. Violet's taxable income will decrease by $100,000 as a result of the IRS adjustment. e. None of the above is correct.

ANSWER: e RATIONALE: $100,000 of salary is reclassified as a dividend. Thus, Violet's taxable income increases by $100,000 because dividends are not deductible. Paula's gross income remains the same. Her salary income decreases by $100,000, but her dividend income increases by $100,000

Hannah, age 70 and single, is claimed as a dependent by her daughter. During 2018, she had interest income of $2,550 and $800 of earned income from babysitting. Hannah's taxable income is: a. $700. b. $900. c. $2,250. d. $2,550. e. None of these.

ANSWER: e RATIONALE: $3,350 gross income - greater of $1,050 or ($800 earned income + $350) - $1,600 (additional standard deduction for age 65 and older) = $600.

Corey is the city sales manager for "RIBS," a national fast food franchise. Every working day, Corey drives his car as follows: Miles Home to office 20 Office to RIBS No. 1 15 RIBS No. 1 to No. 2 18 RIBS No. 2 to No. 3 13 RIBS No. 3 to home 30 Corey renders an adequate accounting to his employer. As a result, Corey's reimburseable mileage is: a. 0 miles. b. 50 miles. c. 66 miles. d. 76 miles. e. None of these.

ANSWER: e RATIONALE: 15 miles + 18 miles + 13 miles = 46 miles. The mileage for driving from his home to the office (20 miles) and from the last worksite to home (30 miles) is not deductible.

Which of the following expenses, if any, qualify as deductible in 2018? a. Contribution to a Roth IRA. b. Costs involved in maintaining an office in the home by a self-employed insurance adjuster. Taxpayer's wife also uses the office as a meeting place for her bridge club. c. Cost of moving to first job location. Taxpayer just graduated from college. d. Job hunting expenses of a fishing guide to become an insurance salesman. e. None of the above

ANSWER: e RATIONALE: A contribution to a Roth IRA is not deductible. An office in the home deduction is not allowed because the location is not used exclusively for business. Moving expenses are not allowed as a deduction from 2018 through 2025. Job hunting expenses are not allowed because a change from fishing guide to insurance salesman is a different trade or business.

in which of the following plans is this statement true: A deduction is allowed for contributions to the plan, and no income tax consequences result from distributions to the participant at retirement. a. Roth IRAs. b. Traditional IRAs. c. Keogh (H.R. 10) plans. d. Both b. and c., but not a. e. None of the above.

ANSWER: e RATIONALE: A deduction is allowed for contributions to traditional IRAs and Keogh (H.R. 10) plans (choices b. and c.) but not to Roth IRAs (choice a.). Distributions are free of income tax in the case of Roth IRAs (choice a.) but not for traditional IRAs and Keogh (H.R. 10) plans (choices b. and c.) Thus, the combination of a deduction and tax-free distributions does not exist (choice e.).

A qualifying child cannot include: a. A nonresident alien. b. A married son who files a joint return. c. A daughter who is away at college. d. A brother who is 28 years of age and disabled. e. A grandmother

ANSWER: e RATIONALE: A grandmother does not meet the relationship test (choice e.). A qualifying child can be a nonresident alien under the adopted child exception (choice a.). The filing of a joint return is not fatal if filing is not required and its purpose is to obtain a tax refund (choice b.). A temporary absence is permissible under the domicile test (choice c.). A brother meets the relationship test, and disability waives the age test (choice d.)

In terms of the tax formula applicable to individual taxpayers, which, if any, of the following statements is correct? a. In arriving at AGI, a taxpayer must elect between claiming deductions for AGI and deductions from AGI. b. In arriving at taxable income, a taxpayer must elect between claiming deductions for AGI and deductions from AGI. c. If a taxpayer has deductions for AGI, the standard deduction is not available. d. In arriving at taxable income, a taxpayer must elect between deductions for AGI and the standard deduction. e. None of these

ANSWER: e RATIONALE: AGI is computed by claiming deductions for and no election is required (choice a.). In arriving at taxable income, deductions for AGI are allowed (choice b.) and the election is between deductions from AGI and the standard deduction (choices c. and d.).

61. Which of the following can be claimed as a deduction for AGI? a. Personal casualty losses. b. Investment interest expenses. c. Medical expenses. d. Property taxes on personal use real estate. e. None of the above.

ANSWER: e RATIONALE: All of these expenses are classified as itemized deductions

Which of the following is not relevant in determining whether an activity is profit-seeking or a hobby? a. Whether the activity is enjoyed by the taxpayer. b. The expertise of the taxpayers or their advisers. c. The time and effort expended. d. The relationship of profits earned and losses incurred. e. All of the above are relevant factors

ANSWER: e RATIONALE: All of these items are relevant factors in determining whether an activity is profitseeking or a hobby.

Which, if any, of the following factors is a characteristic of independent contractor status? a. Services are performed for more than one business. b. Receipt of a Form 1099 reporting payments received. c. Workplace fringe benefits are not available. d. Work-related income and expenses are reported on Schedule C (Form 1040). e. All of the above are characteristic of independent contractor status.

ANSWER: e RATIONALE: All of these might indicate that an individual is an independent contractor.

Which of the following may be deductible in 2018? a. Bribes that relate to a U.S. business. b. Fines paid for violations of the law. c. Interest on a loan used in a hobby. d. All of the above. e. None of the above.

ANSWER: e RATIONALE: Choices a. and b. are not deductible. Interest incurred in connection with a hobby is no longer deductible since 2% miscellaneous itemized deductions are no longer allowed.

Julie was suffering from a viral infection that caused her to miss work for 90 days. During the first 30 days of her absence, she received her regular salary of $8,000 from her employer. For the next 60 days, she received $12,000 under an accident and health insurance policy purchased by her employer. The premiums on the health insurance policy were excluded from her gross income. During the last 30 days, Julie received $6,000 on an income replacement policy she had purchased. Of the $26,000 she received, Julie must include in gross income: a. $0. b. $6,000. c. $8,000. d. $14,000. e. $20,000

ANSWER: e RATIONALE: The $6,000 received under the accident and income replacement policy Julie purchased is excluded from her gross income

Christie sued her former employer for a back injury she suffered on the job in 2018. As a result of the injury, she was partially disabled. In 2019, she received $240,000 for her loss of future income, $160,000 in punitive damages because of the employer's flagrant disregard for the employee's safety, and $15,000 for medical expenses. The medical expenses were deducted on her 2018 return, reducing her taxable income by $12,000. Christie's 2019 gross income from the above is: a. $415,000. b. $412,000. c. $255,000. d. $175,000. e. $172,000.

ANSWER: e RATIONALE: Christie must include in gross income the $160,000 of punitive damages received and the $12,000 for the previously deducted medical expenses. The medical expense recovery is included in gross income under the tax benefit rule.

Which of the following taxpayers may file as a head of household in 2018? Marco provides all the support for his mother, Sienna, who lives by herself in an apartment in Fort Lauderdale. Marco pays the rent and other expenses for the apartment and properly claims his mother as a dependent. Tammy provides over one-half the support for her 18-year old brother, Dan. Dan earned $4,200 in 2018 working at a fast food restaurant and is saving his money to attend college in 2019. Dan lives in Tammy's home. Juan's wife left him late in December of 2017. No legal action was taken and Juan has not heard from her in 2018. Juan supported his 6-year-old son, who lived with him throughout 2018. a. Marco only b. Tammy only c. Juan only d. Marco and Juan only e. Marco, Tammy, and Juan

ANSWER: e RATIONALE: Marco may file as a head of household. His mother is not required to live in his household in order for him to qualify as a head of household. Tammy can claim Dan as a dependent because Dan is a qualifying child and is not subject to the gross income requirement. Juan can file as a head of household under the abandoned spouse rules

Which, if any, of the following is a deduction for AGI? a. State and local sales taxes. b. Interest on home mortgage. c. Charitable contributions. d. Unreimbursed moving expenses of an employee (not in the military). e. None of these.

ANSWER: e RATIONALE: Moving expenses are not deductible (unless individual is in the military). All other items (choices a., b., and c.) are deductions from AGI.

In which, if any, of the following situations is the automatic mileage available? a. A limousine to be rented by the owner for special occasions (e.g., weddings, high school proms). b. The auto belongs to taxpayer's mother. c. One of seven cars used to deliver pizzas. d. MACRS statutory percentage method has been claimed on the automobile. e. None of these.

ANSWER: e RATIONALE: Multiple vehicles (5 or more) cannot qualify (choice c.) nor can vehicles held for hire (choice a.). The car must be owned or leased by the taxpayer (choice b.).

Which of the following is not deductible in 2018? a. Moving expenses in excess of reimbursement. b. Tax return preparation fees of an individual. c. Expenses incurred associated with investments in stocks and bonds. d. Allowable hobby expenses in excess of hobby income. e. All of the above.

ANSWER: e RATIONALE: None of the expenses are deductible in 2018

Aaron is a self-employed practical nurse who works out of his home. He provides nursing care for disabled persons living in their residences. During the day he drives his car as follows. Miles Aaron's home to patient Louise 12 Patient Louise to patient Carl 4 Patient Carl to patient Betty 6 Patient Betty to Aaron's home 10 Aaron's deductible mileage for each workday is: a. 10 miles. b. 12 miles. c. 20 miles. d. 22 miles. e. 32 miles

ANSWER: e RATIONALE: Since Aaron's office is in his home, he has no nondeductible commuting mileage—all of the mileage is deductible.

Tommy, an automobile mechanic employed by an auto dealership, is considering opening a fast food franchise. If Tommy decides not to acquire the fast food franchise, any investigation expenses are: a. A deduction for AGI. b. A deduction from AGI, subject to the 2 percent floor. c. A deduction from AGI, not subject to the 2 percent floor. d. Deductible up to $5,000 in the current year with the balance being amortized over a 180-month period. e. Not deductible

ANSWER: e RATIONALE: Since Tommy is not in a business that is the same as or similar to the one being investigated and did not acquire the new business, his investigation expenses cannot be deducted.

Early in the year, Marion was in an automobile accident during the course of his employment. As a result of the physical injuries he sustained, he received the following payments during the year: Reimbursement of medical expenses Marion paid by a medical insurance policy he purchased $10,000 Damage settlement to replace his lost salary 15,000 What is the amount that Marion must include in gross income for the current year? a. $25,000. b. $15,000. c. $12,500. d. $10,000. e. $0.

ANSWER: e RATIONALE: The medical expenses of $10,000 that were reimbursed by Marion's medical insurance policy can be excluded from gross income. The $15,000 Marion received for the physical personal injury damage settlement can be excluded from gross income even though the payment replaces his salary.

Tax advantages of being self-employed (rather than being an employee) include: a. The self-employment tax is lower than the Social Security tax. b. The overall limitation (50%) on meals does not apply. c. An office in the home deduction (for AGI) is available. d. Job-related expenses are deductions for AGI. e. Both c. and d. are advantages.

ANSWER: e RATIONALE: The self-employment tax is double what the employee pays (choice a.). The overall limitation on meals also applies to self-employed taxpayers (choice b.). Job-related expenses are reported on Schedule C which results in deduction-for-AGI treatment (choice d.). The deduction for office in the home for a self-employed individual is a deduction for AGI.

The § 222 deduction for tuition and related expenses is available: a. Only if the taxpayer itemizes deductions from AGI. b. To deduct that portion of the tuition in excess of that allowed under the lifetime learning credit. c. To cover the tuition of a son who does not qualify as taxpayer's dependent. d. Only if job related. e. None of these.

ANSWER: e RATIONALE: The § 222 deduction is a deduction for AGI (choice a.). The deduction is not available when the lifetime learning credit is used (choice b.). The student involved is not a dependent of the taxpayer (choice c.). The education need not be job related (choice d.).

Robert entertains several of his key clients on January 1 of the current year. Expenses paid by Robert are as follows: Cab fare $ 60 Cover charge at supper club 200 Dinner at club 800 Tips to waiter 160 Presuming proper substantiation, Robert's deduction is: a. $610. b. $640. c. $740. d. $1,220. e. None of these.

ANSWER: e RATIONALE: With the TCJA of 2017, entertainment is no longer deductible

Bob and April own a house at the beach. The house was rented to unrelated parties for 8 weeks during the year. April and the children used the house 12 days for their vacation during the year. After properly dividing the expenses between rental and personal use, it was determined that a loss was incurred as follows: Gross rental income $4,000 Less: Mortgage interest and property taxes $3,500 Other allocated expenses 2,000 (5,500) Net rental loss ($1,500) What is the correct treatment of the rental income and expenses on Bob and April's joint income tax return for the current year assuming the IRS approach is used if applicable? a. A $1,500 loss should be reported. b. Only the mortgage interest and property taxes should be deducted. c. Since the house was used more than 10 days personally by Bob and April, the rental expenses (other than mortgage interest and property taxes) are limited to the gross rental income in excess of deductions for interest and taxes allocated to the rental use. d. Since the house was used less than 50% personally by Bob and April, all expenses allocated to personal use may be deducted. e. Bob and April should include none of the income or expenses related to the beach house in their current year income tax return

a

Sue performs services for Lynn. Regarding this arrangement, use the legend provided to classify each statement. a. Indicates employee status. b. Indicates independent contractor status Lynn determines when the services are to be performed.

a

Sue performs services for Lynn. Regarding this arrangement, use the legend provided to classify each statement. a. Indicates employee status. b. Indicates independent contractor status Sue charges by the hour for her work.

a

Sue performs services for Lynn. Regarding this arrangement, use the legend provided to classify each statement. a. Indicates employee status. b. Indicates independent contractor status Sue does not file a Schedule SE with her Form 1040.

a

Sue performs services for Lynn. Regarding this arrangement, use the legend provided to classify each statement. a. Indicates employee status. b. Indicates independent contractor status Sue was trained by Lynn.

a

Sue performs services for Lynn. Regarding this arrangement, use the legend provided to classify each statement. a. Indicates employee status. b. Indicates independent contractor status. Sue does not work for other parties

a

Under the Swan Company's cafeteria plan, all full-time employees are allowed to select any combination of the benefits below, but the total received by the employee cannot exceed $8,000 a year. I. Group medical and hospitalization insurance for the employee, $3,600 a year. II. Group medical and hospitalization insurance for the employee's spouse and children, $1,200 a year. III. Child-care payments, actual cost but not more than $4,800 a year. IV. Cash required to bring the total of benefits and cash to $8,000. Which of the following statements is true? a. Sam, a full-time employee, selects choices II and III and $2,000 cash. His gross income must include the $2,000. b. Paul, a full-time employee, elects to receive $8,000 cash because his wife's employer provided these same insurance benefits for him. Paul is not required to include the $8,000 in gross income. c. Sue, a full-time employee, elects to receive choices I, II and $3,200 for III. Sue is required to include $3,200 in gross income. d. All of these. e. None of these

a

The alimony recapture rules are intended to: a. Assist former spouses in collecting alimony when the other spouse moves to another state. b. Prevent tax deductions for property divisions. c. Reduce the net cash outflow for the payor. d. Distinguish child support payments from alimony. e. None of these.

b

A scholarship recipient at State University may exclude from gross income the scholarship proceeds used to pay for: a. Only tuition. b. Tuition, books, and supplies. c. Tuition, books, supplies, meals, and lodging. d. Meals and lodging. e. None of these.

b

An employee can exclude from gross income the value of meals provided by his or her employer whenever: a. The meal is not extravagant. b. The meals are provided on the employer's premises for the employer's convenience. c. There are no places to eat near the work location. d. The meals are provided for the convenience of the employee. e. None of these.

b

Sue performs services for Lynn. Regarding this arrangement, use the legend provided to classify each statement. a. Indicates employee status. b. Indicates independent contractor status Sue files a Schedule SE with her Form 1040.

b

Sue performs services for Lynn. Regarding this arrangement, use the legend provided to classify each statement. a. Indicates employee status. b. Indicates independent contractor status Sue has unreimbursed expenses.

b

Sue performs services for Lynn. Regarding this arrangement, use the legend provided to classify each statement. a. Indicates employee status. b. Indicates independent contractor status Sue uses her own helpers.

b

Sue performs services for Lynn. Regarding this arrangement, use the legend provided to classify each statement. a. Indicates employee status. b. Indicates independent contractor status Sue uses her own tools.

b

Sue performs services for Lynn. Regarding this arrangement, use the legend provided to classify each statement. a. Indicates employee status. b. Indicates independent contractor status. The services are performed at Sue's premises.

b

The exclusion of interest on educational savings bonds: a. Applies only to savings bonds owned by the child. b. Applies to parents who purchase bonds for which the proceeds are used for their child's education. c. Means that the child must include the interest in income if the bond is owned by the parent. d. Does apply even if used to pay for room and board. e. None of these.

b

The purpose of the tax rules that apply to below-market loans between family members is to: a. Discourage loans between related parties. b. Prevent shifting of income among family members. c. Prevent gifts from being disguised as bad debt expenses. d. Prevent gift tax avoidance. e. None of these is true.

b

Which of the following is incorrect? a. Alimony is a deduction for AGI. b. The expenses associated with royalty property are a deduction from AGI. c. Contributions to a traditional IRA are a deduction for AGI. d. Property taxes on taxpayer's personal residence are a deduction from AGI e. All of the above are correct.

b

Which of the following statements is correct in connection with the investigation of a business? a. If the taxpayer is not already engaged in the trade or business, the expenses incurred are deductible if the project is abandoned. b. Expenses may be deducted immediately by a taxpayer engaged in a similar trade or business regardless of whether the business being investigated is acquired. c. That business must be related to the taxpayer's present business for any expense ever to be deductible. d. Regardless of whether the taxpayer is already engaged in the trade or business, the expenses must be capitalized and amortized. e. None of the above.

b

Which, if any, of the following statements relating to the standard deduction is correct? a. If a taxpayer dies during the year, his (or her) standard deduction must be prorated. b. If a taxpayer is claimed as a dependent of another, his (or her) additional standard deduction is allowed in full (i.e., no adjustment is necessary). c. If spouses file separate returns, both spouses must claim the standard deduction (rather than itemize their deductions from AGI). d. If a taxpayer is claimed as a dependent of another, no basic standard deduction is allowed. e. None of these.

b RATIONALE: In the case of death, no apportionment is required and the full standard deduction is allowed (choice a.). If married taxpayers file separate returns and one spouse itemizes, the other spouse must also itemize. However, there is no requirement that they each claim the standard deduction—although they may do so (choice c.). A basic standard deduction is allowed for dependents although its determination is subject to special rules (choice d.).

58. Trade or business expenses of a self-employed taxpayer should be treated as: a. Deductible for AGI on Schedule E. b. A deduction from AGI. c. Deductible for AGI on Schedule C. d. An itemized deduction if not reimbursed. e. None of the above

c

Under the alimony rules: a. To determine whether a cash payment is alimony, one must consult the state laws that define alimony. b. A person who receives a property division has experienced an increase in wealth and thus should be subject to tax. c. Alimony paid per a 2015 divorce agreement is included in the gross income of the recipient of the payments. d. A person who earns $90,000 and pays $20,000 in alimony per a divorce agreement entered into in 2019, is allowed to deduct the $20,000. e. None of these.

c

Which of the following are deductions for AGI? a. Mortgage interest on a personal residence. b. Property taxes on a personal residence. c. Mortgage interest on a building used in a business. d. Fines and penalties incurred in a trade or business. e. None of the above

c

Which of the following is not a "trade or business" expense? a. Interest on business indebtedness. b. Property taxes on business property. c. Parking ticket paid on business auto. d. Depreciation on business property. e. All of the above are "trade or business" expenses.

c

Which of the following is not a requirement for an alimony deduction? a. The payments must be in cash. b. The payments must cease upon the death of the payee. c. The payments must extend over at least three years. d. The payor and payee must not live in the same household at the time of the payments. e. All of these are requirements for an alimony deduction

c

Assuming a taxpayer qualifies for the exclusion treatment, the interest income on educational savings bonds: a. Is gross income to the person who purchased the bond in the year the interest is earned. b. Is gross income to the student in the year the interest is earned. c. Is included in the student's gross income in the year the savings bonds are sold or redeemed to pay educational expenses. d. Is not included in anyone's gross income if the proceeds are used to pay college tuition. e. None of these

d

Cash received by an employee from an employer: a. Is not included in gross income if it was not earned. b. Is not taxable unless the payor is legally obligated to make the payment. c. Must always be included in gross income. d. May be included in gross income although the payor is not legally obligated to make the payment. e. None of these

d

During 2017, the first year of operations, Silver, Inc., pays salaries of $175,000. At the end of the year, employees have earned salaries of $20,000, which are not paid by Silver until early in 2018. What is the amount of the deduction for salary expense? a. If Silver uses the cash method, $175,000 in 2017 and $0 in 2018. b. If Silver uses the cash method, $0 in 2017 and $195,000 in 2018. c. If Silver uses the accrual method, $175,000 in 2017 and $20,000 in 2018. d. If Silver uses the accrual method, $195,000 in 2017 and $0 in 2018. e. None of the above is correct.

d

The alimony rules applicable to divorces entered into before 2019: a. Are based on the principle that the person who earns the income should pay the tax. b. Permit tax deductions for property divisions. c. Look to state law to determine the definition of alimony. d. Distinguish child support payments from alimony. e. None of these.

d

The exclusion for health insurance premiums paid by the employer applies to: a. Only current employees and their spouses. b. Only current employees and their spouses and dependents. c. Only current employees and their disabled spouses. d. Current employees, retired former employees, and their spouses and dependents. e. None of these.

d

Which of the following is a required test for the deduction of a business expense? a. Ordinary b. Necessary c. Reasonable d. All of the above e. None of the above

d

For a president of a publicly held corporation hired in 2018, which of the following are not subject to the $1 million limit on executive compensation? a. Contribution to medical insurance plan. b. Contribution to pension plan. c. Premiums on group term life insurance of $50,000. d. Only b. and c. are not subject to the limit. e. a., b., and c., are not subject to the limit.

e

Which of the following is correct? a. A personal casualty loss incurred from a Presidentially declared disaster is classified as a deduction from AGI. b. Real estate taxes on a taxpayer's personal residence are classified as deductions from AGI. c. An expense associated with rental property is classified as a deduction for AGI. d. Only a. and b. are correct. e. a., b., and c., are correct.

e


Conjuntos de estudio relacionados

Maternal Newborn Exam One- ALL MATERIAL!

View Set

Chapter 8 : Genome structure, chromatin and the nucleosome. (Watson)

View Set

Chapter 59: Assessment and Management of Patients With Male Reproductive Disorders

View Set

NEC 430 Review, ELAP 1032 Spring

View Set

Evaluate the expression when n = 4

View Set

Constitution (Separation of Powers)

View Set